Project Planning & Design (PPD)J

Ace your homework & exams now with Quizwiz!

How far do Timber piles extend into the air?

110 Feet. They are also the weakest type of pile and would not e able to support the loads for a 15-story office building.

How depp is a W12x120 wide flange beam?

12"

How far can drilled piers with bells extend into the air?

120 Feet into the earth with relatively low bearing capacity.

What is the coltage for a single familay house?

120 volt / 240 volt, single phase, 3-wire

A 3,600-square-foot one-story cafe and market is to have a mezzanine at the rear of the space. Based on the IBC, what can be the maximum size of the mezzanine in square feet?

1200 SF Based on IBC 2018 Section 505.2.1, the maximum area of a mezzanine is 1/3 of the size of the room in which it is located. The maximum size for the mezzanine can be calculated with the following equation: 3,600 sq. ft. x 1/3 = 1,200 sq. ft.

How many amps for a 2160-watt bedroom circuit in a single-family detached house? Ignore the power factor. Given W=I*V

18amps Standard voltage is 120 volts.

The architect is establishing the orientation for a roughly rectangular building in a hot-humid climate region. The goal is to minimize solar radiation from the east and west while taking advantage of the heating potential of the south facade in the winter. What is the optimum orientation angle for the architect to use for the east-west datum?

5 degrees off south is ideal for a hot-humid climate. 17.5 degrees off south is ideal for a temperate climate 25 degrees off south is ideal for a hot arid climate 12 degrees off south i ideal for a cool climate

The first modern zoning code was introduced in New York City in 1916. Which exemplifies the purpose of zoning?

-Protect access to direct sunlight and fresh air -Ensure incompatible types of building uses do not disturb residents Zoning is centered on protecting the rights of residents and property owners. This includes disturbances such as putting a school next to a noisy factory -Protect local residents from undesirable types of business This concept is in place in order to group compatible businesses and ensure the wrong kinds of businesses don't end up too close to each other. -Protect the amount of open space for local residents. Determine the allowable function of a building The allowable function of a building falls under building rather than zoning code. Building codes comprise a set of rules that specify minimal quality and safety requirements. Regulate the number of people or occupant loads of a building or structure Occupancy falls under building rather than zoning code. Building codes

What is a raft foundation?

Also called a mat foundation, is essentially a continuous slab resting on the soil that extends over the entire footprint of the building, thereby supporting the building and transferring its weight to the ground.

An architect is working on a kitchen renovation for a high-end residence and needs to specify a wood species for the cabinets. The client has requested these to be made of solid wood in lieu of a lower-quality laminate. What of the following wood species should the architect recommend?

Birch Correct. Hardwoods are typically used in finish work, and birch is a type of hardwood.

An architect is working on a kitchen renovation for a high-end residence, and needs to specify a wood species for the cabinets. The client has requested these to be made of solid wood in lieu of a lower quality laminate. What of the following wood species should the architect recommend?

Birch Correct. Hardwoods are typically used in finish work, and birch is a type of hardwood. Douglas fir Incorrect. For fine furniture and interior finish details, hardwoods are often chosen. Douglas fir is a softwood.

Different pattern matching of wood veener?

Book Matching Slip Matching Random Matching Pleasing Matching

An architect is working on the design of a new office facility. The project requires sustainable materials and practices, so the architect is performing a life cycle analysis on the building envelope systems. Which of the following tools provides a life cycle analysis using materials and values specific to the United States?

Building for Environmental and Economic Sustainability (BEES) BEES provides environmental and economic data for products and is specific to the United States. Note: Environmental Product Declarations (EPD) are based on life cycle analysis data, and are not part of the analysis itself.

An architect is designing a multistory office building. Which design strategy represents a successful integration of a building system with the project design?

A light shelf along the south facade Correct. Building integration is the organized consideration of the interrelationship between the productive systems of a facility for the purpose of optimizing the effectiveness of the collection of systems. A light shelf represents a successful integration of the building system since it integrates well with the building lighting, glazing and HVAC systems. It reflects light deeper into the building, thereby providing more light in the interior. It blocks excessive light and heat from entering spaces close to the perimeter wall. A "sandwich" system for the ceiling plenum Incorrect. Most multistory commercial buildings are very wasteful of the space above the finished ceiling and below the floor of the level above. This is the "sandwich" volume for various building systems such as structures, HVAC, electrical, communications, plumbing, and fire protection. To make the design process simpler, each system is given its own layer, allocating an independent volume to each of the systems. The design process thereby becomes more like volume allocation, rather than building system integration.

U-value

A measure of thermal transmittance, which is defined as the rate of heat transfer through an assembly divided by the difference in temperature across that assembly. Found by taking the inverse of the R-value of that assembly. A lower value indicates a higher insulation factor.

What is intumescent paint?

A paint used for fire retardation. The paint bubbles, swells or expands to form a protective coating when exposed to extreme heat. (expensive)

wht are site photometricts?

A photometric plan (aka photometric analysis, photometric lighting study) is a digital survey of your site with a proposed lighting solution. This light study allows you to view the lighting level at the job-site before construction begins.Normally conducted by an electrical enginner

What is a plenum?

A plenum space is a part of a building that can facilitate air circulation for heating and air conditioning systems, by providing pathways for either heated/conditioned or return airflows, usually at greater than atmospheric pressure

Generally, which offers more resistance to lateral forces? Rigid core or Rigid Perimeter

A rigid perimeter

Which configuration is the least expensive? -A moment Resisting Frame -Shear Wall -Braced

A shear wall

The following plumbing riser diagram has been included in the drawing set and the architect is reviewing it for coordination. Drag the correct labels on the left and place them in their boxes on the right to correctly label the elements in the plumbing riser diagram.

A vent stack is a vertical pipe that connects all the plumbing fixture vents and exhausts through the roof. A soil stack is a vertical pipe that connects all the soil and waste lines and empties to the sewer or septic tank. A cleanout is a convenient location in a plumbing system that allows the pipes to be cleaned or unclogged if necessary. A trap is the U-shaped pipe at plumbing fixtures that prevents sewer gases from entering a building.

A firm is preparing a design presentation for a client during the early schematic design phase. The principal of the firm wants to be able to quickly perform a large number of massing and facade studies so the firm can choose their favorites to present. Which of the following approaches would allow the firm's staff to produce the most studies in the least amount of time?

Building information modeling (BIM) Correct. While BIM may require a bit of time when setting up the modeling parameters for a project, it enables the relatively quick study of a variety of massing and facade alternatives. Physical site model with massing studies Incorrect. Physical models are time-consuming, particularly if facade elements are also to be included. Physically modeling a site also takes a large amount of time, whereas modeling a site with BIM software can be done relatively quickly.

An architect is designing the building shell of a two-story medical office building and is tasked with coordinating tenant build-outs with each tenant's design team. One of the tenants on the first floor will be installing a piece of medical equipment that will require a revision to the electrical power supply for the building. Who would be responsible for making sure this revision is properly coordinated?

Building shell architect Correct. The building shell architect is ultimately responsible for making sure that the electrical revisions have been coordinated properly based upon the information provided by their own electrical engineer and the tenant's design team. Tenant's electrical engineer Incorrect. The tenant's electrical engineer would do the electrical design for the build-out and provide that information for the building shell architect and electrical engineer to review and coordinate with the building's power supply.

What is bituminous roof made of?

Built-Up Roof (BUR) Asphalt Coal Tar Modified Bitumen APP Atactic Polypropylene SBS styrene-butadiene-styrene

What do we use geotextiles for?

Geotextiles, which may look like a woven mesh or like a plastic tarp, can be buried to strengthen soil, stabilize soil, separate different strata of soil, prevent erosion, keep out weeds, keep water out, allow water to filter through, or seal soil to keep contaminants in.

Which is larger a Beam or a Girder?

Girders

What is Chromatic Glass?

Glass that can change its optical properties, for example by getting darker in either warmth or sunlight.

In a design competition for a nature center, the requirements specify that natural materials must be prioritized. The nature center will include an outdoor bird blind. Which type of wood is most suitable for the outdoor bird blind?

Cedar Correct. Cedar is a heartwood which is best in exterior conditions due to natural resistance to the elements.

Where should the architect look to find the requirements for natural lighting in the International Building Code (IBC)?

Chapter 12 "Interior Environment" Information about the minimum light and air requirements are found in section 1204 Lighting.

A developer approaches an architect about a possible hotel project where they would like to provide doors and windows that face a courtyard. They would also like these doors and windows to meet the minimum code requirements for natural lighting. Where should the architect look to find the requirements for natural lighting in the International Building Code (IBC)?

Chapter 12 Correct. Chapter 12 is "Interior Environment." Information about light and air requirements are found in Section 1204 Lighting.

Which is stronger? Mild (hot) Steel or Cold Rolled Steel

Cold-rolled Steel Cold rolled steel is essentially hot rolled steel that has been further processed by being allowed to cool at room temperature and annealed or temper rolled. Cold rolling produces steel with closer dimensional tolerances and a wider range of surface finishes than hot rolling.

What is a one-way slab?

In a one-way slab, the bending moment occurs in a singular direction, while in a two-way slab it occurs in both directions. Another important difference is the content of steel in both types of slabs. One-way slabs have less content of steel than the two-way slab, as it has steel in one direction. max span= 12feet

What is the maximum offset of a column on a two-way slab?

In a two-way slab system, the column offset can't be anymore than 1/10 of the horizontal dimension. I

During the preliminary review of column layout for a cast-in-place, two-way slab, the architect sees the plumbing stacks are in conflict with a column location. In order to resolve the conflict, the architect suggests shifting the columns in question off the main gridline. What is the maximum distance the architect can propose the column be moved? Using the image below, drag the correct value onto the dimension string. Round your answer to the nearest tenth of an inch.

In a two-way slab system, the column offset can't be anymore than 1/10 of the horizontal dimension. In this case, the horizontal dimension is 26'; therefore, the offset can be up to 2'7"

An architect has just entered into a contract with a new client. The architect is explaining to their team which design alternative studies are included in the scope of basic services and which should be performed as an additional service. Which of the following design alternative studies are covered under the scope of the architect's basic services, per the AIA Standard Form of Agreement between Owner and Architect?

Inclusion of design alternates in specifications Correct. Per AIA B101-2017 Article 4.2.1.6, the architect shall include design alternatives for bidding purposes in the contract documents, which include the specifications. Sustainable design alternatives Correct. Per AIA B101-2017 Article 3.2.5.1, the architect shall consider sustainable design alternatives as part of the basic services. Alternative materials Correct. Per AIA B101-2017 Article 3.2.5.2, the architect shall consider alternative materials as part of the basic services. Alternate building systems Correct. Per AIA B101-2017 Article 3.2.5.2, the architect shall consider alternative building systems as part of the basic services. Multiple preliminary designs Incorrect. Per AIA B101-2017, this is listed as an additional service.

A structural engineer has told an architect that the total pressure on a retaining wall cannot exceed 1500 plf. The concrete wall has a total length of 40 ft and is adjacent to a parking lot. The pressure on the wall has been calculated to be 30 lbs per cubic foot. Use the formula P = 0.5 (p) h^2. How high can the architect make the wall?

Incorrect - the answer is 10 We will use the formula P = 0.5 (p) h^2, where the variables are defined as follows:P = total pressure (1500 plf)p = media pressure (30 lbs/ft^3)h = heightPlugging in the numbers that are known, we get:1500 plf = 0.5 (30 lbs/ft^3) (h^2)Using algebra to solve for h, we find the answer:h = 10 ft

An architect is preparing construction documents and deciding which species of wood to specify, for various items included in their design. Which properties of soft wood make the architect more likely to specify it for use in framing than as finished carpentry?

Inexpensive Correct. Softwood is plentiful and therefore inexpensive. Coarse grain pattern Correct. The coarse grain pattern of softwoods makes it less attractive as a finish and better suited as hidden structural elements. Aesthetically uninteresting Correct. The lack of aesthetic appeal makes softwood a better option for hidden structural elements than for finishes. Greater fire resistance Incorrect. Hardwoods have a greater resistance to fire than softwoods do.

An architect is looking to build a new multifamily apartment complex in an urban center. Space is limited, so the architect is looking to revitalize the area by making efficient use of a 5,000-square-foot vacant lot. The lot is near existing transit infrastructure, and all required infrastructure is in place for a new development. Which of the following development strategies would be the most appropriate for this project?

Infill development Correct. An infill development would be most appropriate for this project, as this type of development takes advantage of vacant or undeveloped properties while utilizing the existing infrastructure of the neighborhood.

An architect is looking to build a new multifamily apartment complex in an urban center. Space is limited, so the architect is looking to revitalize the area by making efficient use of a 5,000-square-foot vacant lot. The lot is near existing transit infrastructure, and all required infrastructure is in place for a new development. Which of the following development strategies would be the most appropriate for this project?

Infill development Correct. An infill development would be most appropriate for this project, as this type of development takes advantage of vacant or undeveloped properties while utilizing the existing infrastructure of the neighborhood. Transit-oriented development Incorrect. While a transit-oriented development takes advantage of existing infrastructure, this type of development is typically much larger than this application. Infill development would be more appropriate.

The impervious area of the project site is 73 percent. Local code now requires a minimum of 50 percent pervious surfaces to minimize stormwater runoff and combined sewer overflow issues. Which solutions should the architect explore?

Install a green roof Correct. A green roof installation is an accepted solution for reducing stormwater runoff. It also adds insulation to the roof, sequesters carbon, and filters air pollution. Replace portions of the parking lot with pervious pavement Correct. Adding pervious pavement surfaces in the parking lot will reduce stormwater runoff. Plant trees Correct. Planting trees will help collect and store rainfall in addition to providing shade (reducing heat island effect), and improving guest experience. Install underground stormwater storage below the parking lot Correct. Installation of underground stormwater collection and storage tanks beneath the parking lot is an accepted solution, especially for a site with 73 percent impervious surface areas that are mostly for parking. Reduce parking lot surface Incorrect. The building classification code will determine parking lot size, so reduction of spaces may not be an option.

What is one of the major disadvantages if a cental HVAC all-water system?

It provides no humidification. On the contrary, it provides good dehumidification. So this system will be suitable for Hot-humid climates only

Glulam

Short for glue-laminated structural lumber, produced by joining flat strips of wood with glue. You can add a High tensile strength laminate for additional strong ablong 40% reduction.

An architect is working with a structural engineer on the design of a 1-story masonry retail building. Which of the following describes this building's likely fundamental period?

Short fundamental period Correct. The shorter and stiffer a structure or object, the shorter its fundamental period Taller building have longer fundamental periods

An architect is researching methods to protect fasteners from corrosion. Which of the following types of metal fasteners would provide the best cathodic protection?

Stainless steel Correct. Stainless steel is the most cathodic of these options, and it is made so by adding chromium directly to molten steel. Galvanized steel Incorrect. While the galvanized coating of steel is considered sacrificial as it oxidizes, the steel beneath this layer is not as cathodic as stainless steel.

An architect is researching methods to protect fasteners from corrosion. Which of the following types of metal fasteners would provide the best cathodic protection?

Stainless steel\ Correct. Stainless steel is the most cathodic of these options, and it is made so by adding chromium directly to molten steel. Galvanized steel Incorrect. While the galvanized coating of steel is considered sacrificial as it oxidizes, the steel beneath this layer is not as cathodic as stainless steel.

An architect is designing an office building and the top floor of the building will be more than 30 feet above the lowest level of fire department vehicle access. Which fire protection system is required by the International Building Code for this condition?

Standpipes Correct. Buildings where the floor level of the highest story is located more than 30 feet above the lowest level of fire department access shall be equipped with a standpipe. Annunciator panel Incorrect. An annunciator panel is a unit which contains displays and equipment to provide information on the condition of a building. It does not relate to water supply or range of extinguishing potential, which is what the question is asking.

A client has enlisted the help of an architect to design a house in the hot, arid climate of Phoenix, Arizona. The architect is considering how to design the building to provide maximum thermal comfort for the occupants in this climate. What strategy will offer the most thermal comfort?

Providing shade from the sun Correct. Shade is important in all hot climates, whether humid or arid. Incorporating large openings for ventilation Incorrect. Large openings are more appropriate for a hot-humid climate, where breezes are a top priority. Designing a light and airy structure Incorrect. A light and airy structure is appropriate for a hot-humid climate where breezes, cross ventilation, and not storing heat are the top priorities. Buildings in hot-arid climates should maximize thermal mas

What is Water-to-water system?

Pumped water over the condenser and a different pumped water system over the evaporator.

An architect is working on an urban multi-family housing project and aims to limit the heat island effect. Which surface has the highest albedo?

White painted surface Correct. White paint has an albedo of 0.5 - 0.9. Albedo is the measure of a surface's solar reflectivity, from 0 to 1. An albedo of 0 would absorb all solar radiation and an albedo of 1 would reflect all solar radiation. Studies have shown that the cooling loads of a building and the heat island effect of the area can be reduced by changing the roof and walls to a material with a high albedo.

wood-plastic composite (WPC)

Wood-like products made from wood fibers, plastics of various types, and other additives, with a plastic content not exceeding 50 percent

What is a two way slab?

The slabs which are supported directly on the column without beams or girders are called flat slabs.

What means infill development?

The term "infill development" refers to building within unused and underutilized lands within existing development patterns, typically but not exclusively in urban areas.

What steel structural members ar not required to be fireproofed?

Those that resist lateral forces only. (and that is their only hob). Ex: Bracing, needn't be fireproofed.

An architect is working with a developer to build single-family homes that are in the flight path of a nearby airport. Which of the following items would offer the best opportunities to mitigate noise in the houses coming from airport operations?

Triple-pane windows Correct. Triple-pane windows add extra air spaces between panes of glass that can help mitigate airport noise. Central air conditioning and heating Correct. A key factor in airport noise mitigation is keeping windows and doors closed. Central air conditioning and heating will provide year-round fresh air ventilation while allowing the windows to be closed. Baffled vents Correct. Airport noise can enter the home through exterior vents in the house, such as exhaust fans and crawl space vents. Providing sound baffles in these vents can help mitigate the noise. Exterior wallboard with resilient fasteners Correct. Attaching wallboard to exterior wall studs with resilient fasteners or clips reduces sound transmission through the walls. Reduced air space in exterior walls Incorrect. To mitigate sound through exterior walls, it is better to increase the air space in the wall.

A decorative, non-load-bearing masonry wall is being constructed in the lobby of an office building. This wall will be used as a thermal mass to passively heat the lobby. Which type of mortar should the architect specify for this application?

Type O Correct. Type O is a low- to medium-strength mortar and can only be used for non-load-bearing walls. It is the most cost-effective choice. Type M Incorrect. Type M is a high-strength mortar used for severe weather exposures. Type N Incorrect. While Type N could be used, it is not as cost effective as type O. Type S Incorrect. Type S is a medium- to high-strength mortar, and is commonly used in structural walls.

A developer has asked an architect to propose an overall layout for their new shopping development. Since the site is small and hilly, the developer wants to minimize the impact on the existing site by reducing the amount of impermeable area, while maximizing the number of parking spaces near each storefront. Which of the following building layouts will meet all of the developer's needs?

U-shaped layout with storefronts along the curve Correct. A U-shaped layout provides a compact parking surface with a minimal amount of paving and a smaller building footprint, both of which reduce the overall impact on the site. Small squares distributed around the site Incorrect. Smaller building footprints might allow for less site disturbance for the buildings, but distributing them throughout the site would increase the need for access roads and single-business parking areas.

An architect is designing a new boat house by a lake. The site slopes down from the boat house towards the lake with a significant 2' drop in one location that is being shored through the use of a 2' high brick retaining wall. Which of the following components may be omitted from the design of the retaining wall?

Weep holes and drainage pipe Correct. For retaining walls 2'-0" tall or less, seep holes and drainage pipes are not required. Note: Concrete footing located below the frost line Incorrect. Despite the wall being diminutive, a concrete footing would still be required to resist the overturning force of the soils above.

An architect is calculating the annual depreciation value for a geothermal system. The system costs $22,000 to install and has an expected useful life of 35 years. The system has a salvage value of $1,500 and requires maintenance every 5 years, at $500 per scheduled maintenance appointment. What is the annual depreciation value of the geothermal system? Round to the nearest dollar.

the answer is 586 First, subtract the salvage value from the initial cost: $22,000 - $1,500 = $20,500 Then, divide by the useful life expectancy of the system: $20,500 / 35 years = $585.71, which rounds to $586. The maintenance costs are irrelevant to annual depreciation calculations.

What are some of the elements of Universal Desgin?

-Equitable Use. The design is useful and marketable to -people with diverse abilities. ... -Flexibility in Use. ... -Simple and Intuitive Use. ... -Perceptible Information. ... -Tolerance for Error. ... -Low Physical Effort. ... -Appropriate Size and Space for Approach and Use.

An architect is creating a hotel design for a hospitality client with properties in four distinctly different climates. The client would like to use a single design for all the locations while utilizing natural light to reduce energy use. Which daylighting strategies should the architect implement in all climate regions to improve energy efficiency?

-Maintain on-site daylight access -Manage glare by specifying tinted glass Mitigating glare ensures occupants will refrain from using blinds Tom completely block daylight. -Utilize reflection and incorporate diffused light to promote daylighting deeper into the building core -Modify the landscape design to use planting to modulate and filter sunlight based on each location. Providing active heating only is used in cold climates. Therefore, is incorrect.

What is the minimum slope of the building sewer?

.5% to .2% depending on the size of the pipe. A greater slope is required for smaller pipes.

What is the diameter of #8 reinforcing bar?

1" We measure rebar in 8th of an ich8/8=1"

What is the minimum required fire-rated separation between the private garages?

1.0 Hour THE IBC requires a 1-hour barrier between the garages

An existing warehouse is being divided up into luxury apartments. The lowest level will serve as parking for the building, and will be divided up into private garages. What is the minimum required fire-rated separation between the private garages?

1.0-hour Correct. The International Building Code (2018) 406.3.1 requires a 1-hour fire barrier between the garages.

About how deep should a steel girder be?

1/15th of the span THe width is about 1/3 of the depth

How depth should a open web steel be?

1/20 the span

What is the recommended slope for drain pipes?

1/4" per foot.

For an attic area of 1,500 square feet, what is the total required area of vents?

10 SF

How far do Precast concrete piles extend into the air?

100 feet into the air with relatively low bearing capacity

What is 1 kip?

1000 Lbs per sq in

What is the bending Moment (M) of the Beam Below?

1000 ft-lbs (In the moment, the beam wants to smile)

During the design development phase for a small office building in Montana, the architect determines that R-19 batt insulation is required for the exterior walls. The architect is looking to achieve the minimum requirement as efficiently as possible, and wants to use dense-pack cellulose batt insulation, which has an R-Value of 3.8 per inch. What common size wood stud should the architect specify to provide R-19 using the preferred insulation in the cavities between exterior wall studs?

2"x6" Correct. The problem states that cellulose batts are R 3.8 per inch. 19/3.8 = 5." Since a minimum cavity of 5" is required, the most appropriate commonly available dimensional lumber is the 2x6, which measures 1.5" x 5.5." If the entire cavity is filled with cellulose batt, then this assembly would provide for R-20.9 (5.5 x 3.8), which exceeds the minimum requirement. 2"x5" Incorrect. While 5" is the minimum depth that will fit the required insulation, 2x5 is not a common lumber size.

What is concrete span?

20'-30'

What is stell span?

25'-35'

What is the voltage run for large buildings?

277 olt/480 volt, 3-phase, 4-wire (for large buildings) There are significant savings for big buildings with higher voltages (less copper). Smalleer wires

For a building with 277/480 volt, three-phase, four-wire service, typically the electric lighting runs on ____________

277 volts

What is the drying time for concrete?

28Days

An architecture firm is designing a 21-story office building in a metropolitan area. Ten elevators will be provided, based on the amount of occupants calculated to use the space. What is the minimum number of hoistways that can be used?

3 hoistways Correct. Each hoistway can contain four elevators at maximum.

What is the minimum recommended width for a buffer between the development and the river to preserve wildlife habitat?

300'-0"

According to the 2010 ADA guidelines, what is the required minimum ramp width, measured between the handrails?

36" An accessible route is required to be a minimum of 36" wide with width reductions permitted at certain instances long the route. Note: While doorways and certain other instances along an accessible route may be reduced to 32", the route itself must maintain a minimum width of 36"

A ramp leading to an existing building forms a portion of an accessible route. According to the 2010 ADA guidelines, what is the required minimum ramp width, measured between the handrails?

36" Correct. An accessible route is required to be a minimum of 36" wide, with width reductions permitted at certain instances along the route. Refer to the 2010 ADA Standards for Accessible Design, Section 405.5.

If the concrete beam its one foot wide, how deep should it be?

36" Like steel, there is a 3 to 1 ratio. W x 3 = L

About how deep should a steel beam spanning 60 feet be?

3ft Depth = 1/20 the Span therefor, 1/20x60ft=3ft

One acre feet?

43560 sf

An architect is working on a commercial project located in South Florida, where the total heating load is 60,000 BTU/h. What is the required capacity of the refrigerant cooling system? Provide your answer in tons.

5 One ton of cooling is equal to 12,000 BTU/h. 60,000/12,000 = 5 tons.

The architect is establishing the orientation for a roughly rectangular building in a hot-humid climatic region. The goal is to minimize solar radiation from the east and west while taking advantage of the heating potential of the south facade in the winter. What is the optimum orientation angle for the architect to use for the east-west datum?

5 degrees off south Correct. Five degrees off south is ideal for a hot-humid climate.

The architect is establishing the orientation for a roughly rectangular building in a hot-humid climatic region. The goal is to minimize solar radiation from the east and west while taking advantage of the heating potential of the south facade in the winter. What is the optimum orientation angle for the architect to use for the east-west datum?

5 degrees off south Correct. Five degrees off south is ideal for a hot-humid climate. 17.5 degrees off south Incorrect. 17.5 degrees off south is ideal for a temperate climate. 12 degrees off south Incorrect. Twelve degrees off south is ideal for a cool climate. 25 degrees off south Incorrect. Twenty-five degrees off south is ideal for a hot-arid climate.

The architect is establishing the orientation for a roughly rectangular building in a hot-humid climatic region. The goal is to minimize solar radiation from the east and west while taking advantage of the heating potential of the south facade in the winter. What is the optimum orientation angle for the architect to use for the east-west datum?

5 degrees off south Correct. Five degrees off south is ideal for a hot-humid climate. 25 degrees off south Incorrect. Twenty-five degrees off south is ideal for a hot-arid climate. 12 degrees off south Incorrect. Twelve degrees off south is ideal for a cool climate. 17.5 degrees off south Incorrect. 17.5 degrees off south is ideal for a temperate climate.

An architect is designing a single-family residence. To minimize plumbing line lengths, the architect chooses to align the following bathroom fixtures on the same wall: A 30"-wide lavatory with vanity A 30" x 60" tub and shower A toilet Which size is the most efficient layout for this bathroom?

5' X 8' A 5' x 8' bathroom is the most efficient of the options. In inches, the bathroom would be 60" x 96". The toilet (with 36" of space), the 30-inch lavatory and vanity, and the 30-inch side of the tub and shower would all be placed against one wall, as required. The tub and shower would extend down the 60-inch wall. Topic: Project Integration of Program & Systems

How many stories can a stell building be id the steel structure is uprotected from fire?

5-Stories If you fireproof the steel there is really no limit on how height you can build.

An architect is reviewing plans for a townhome development. The homes have on-grade garages with bedrooms above on the second floor. The construction type will be V-A with an NFPA 13R sprinkler system. What is the minimum required separation between the garage and the bedrooms above?

5/8" + 1/2" gyp board applied to the structure above the garage that supports the dwelling unit Correct. Per IBC 2018 section 406.3.2.1: "Garages beneath habitable rooms shall be separated from all habitable rooms above by not less than a 5/8-inch (15.9 mm) Type X gypsum board or equivalent and 1/2-inch (12.7 mm) gypsum board applied to structures supporting the separation from habitable rooms above the garage." 6" studs + 5/8" gyp applied to the garage ceiling Incorrect. The gypsum board must be applied to the structure. A gypsum ceiling would not meet the International Building Code requirements for the horizontal separation.

An architect is assessing ADA compliance of an existing elevator in a multi-story residential building. What is the minimum depth and width of the elevator car required by the ADA?

54" x 36" Correct. Per ADA Standard 407.4.1 Car Dimensions, existing elevator car configurations that provide an inside clear depth of 54 inches (1370 mm) minimum and a clear width of 36 inches (915 mm) minimum shall be permitted.

How deep should the bottom of a spread footing be in a climate with a frost depth of 60"?

60" If you don't go deep enough, freeze-thaw cycles will heave your foundation.

An architect is designing a U-shaped kitchen for an accessible unit in an apartment building. The architect laid out the kitchen cabinets on two of the walls and the refrigerator on the third wall. What is the minimum clear space between the faces of the cabinets and refrigerator?

60" Correct. 60" minimum is required, so that a wheelchair user can turn around in the kitchen. The 60" requirement is found in both ANSI ICC A117.1-2009 and the 2010 ADA standards. 40" Incorrect. 40" clear between cabinets and the fridge is what is required by the Fair Housing Act. 40" would work if the layout were a galley kitchen.

A developer is looking to build a three-story office building with a total of 72,000 square feet in a suburban area. They are working with an architect to evaluate potential sites for the project. Approximately what percentage of the site should be allocated for surface parking and landscaping?

75% Correct. For an office building of this size on a typical suburban lot, approximately 25% of the lot area should be allocated for the building, with around 50% for parking and 25% for landscaping (for a total of 75%).

A developer is looking to build a three-story office building with a total of 72,000 square feet in a suburban area. They are working with an architect to evaluate potential sites for the project. Approximately what percentage of the site should be allocated for surface parking and landscaping?

75% Correct. For an office building of this size on a typical suburban lot, approximately 25% of the lot area should be allocated for the building, with around 50% for parking and 25% for landscaping (for a total of 75%).

The materials for the exterior wall of a building are required to meet strict energy codes for insulation. Which material selection has the best insulation value?

8 inches concrete: k-factor= 10 Correct. The k-factor of a material is a measure of thermal conductivity and not thermal resistance. Thermal resistance is measured by a material's R-value. The highter the R-value, the better the thermal insulation. The K-factor can be determined by taking the thickness of the insulation and dividing it by the R-value. In this example, K-factor = 10 10=8 inches/R 10R=8 R=.8

What is an arboretum?

A botanical garden devoted to trees

Air Handling Unit (AHU)

A device used to condition and circulate air as part of a heating, ventilating, and air-conditioning (HVAC) system.

Dual smoke detectors

A fire alarm system which requires a minimum of two separate smoke detectors to be triggered to sound an alarm.

What is a two way flat plate?

A flat plate is a two-way system usually supported directly on columns or load-bearing walls. The main feature of the flat plate floor is a uniform thickness with a flat soffit which requires only simple formwork and is easy to construct. < 25' span

What is a flitch beam

A flitch beam is a series of one or more steel plates bolted in between wood structural members.

Purlin

A horizontal structural element which, in conjunction with rafters, supports a roof.

Chillers

A system that cools water to be circulated throughout the building to air handling units and fan coil units. A machine that removes heat from a liquid via a refrigeration cycle. In many commercial facilities that require air conditioning, chillers are a major energy user. This, it is important to select a chiller that costs as little as possible to operate for the specific application.

Large quantities of calcium and magnesium deposits have recently been found in the domestic water supply of an educational facility. When these chemicals are found in the water in large quantities, the water is considered hard, causing scale and build-up in the plumbing system. Which of the following processes can be implemented to correct this? Check the three that apply.

Aeration Correct. Aeration is used to improve the taste and color of water and also help remove iron and manganese. Reverse osmosis Correct. Reverse osmosis is effective at removing calcium and magnesium ions that make water hard. Ion-exchanger Correct. An ion-exchanger pulls the calcium and magnesium out of the water.

What type of admixture should the architect specify to improve the resistance of the cured product to cracking induced by freeze-thaw cycles?

Air-entraining agents Air-entraining agents disperse air in the form of small air bubbles throughout the concrete, Portland cement paste, or mortar. This has a number of benefits to the cued product when used properly, including improved resistance to frost attack and deterioration. Note: "Surfactants" helps the cement particles and other additives in the mix to emulsify and disperse evenly.

An architect is designing a large outdoor pavilion in a cool climate with cast-in-place concrete as the primary construction material. What type of admixture should the architect specify to improve the resistance of the cured product to cracking induced by freeze-thaw cycles?

Air-entraining agents Correct. Air-entraining agents disperse air in the form of small air bubbles throughout concrete, Portland cement paste, or mortar. This has a number of benefits to the cured product when used properly, including improved resistance to frost attack and deterioration. Superplasticizers Incorrect. Superplasticizers improve the workability of a concrete or mortar mix at a given water-cement ratio. They can also be used to reduce the amount of mixing water needed for a desired workability.

An architect is designing a large outdoor pavilion in a cool climate with cast-in-place concrete as the primary construction material. What type of admixture should the architect specify to improve the resistance of the cured product to cracking induced by freeze-thaw cycles?

Air-entraining agents Correct. Air-entraining agents disperse air in the form of small air bubbles throughout the concrete, Portland cement paste, or mortar. This has a number of benefits to the cured product when used properly, including improved resistance to frost attack and deterioration. Surfactants Incorrect. Surfactants reduce the surface tension of the water in a concrete or mortar mix. This helps the cement particles and other additives in the mix to emulsify and disperse evenly.

A good barrier for preventing sound from transmitting from one room to the other is___ Absorptive or Airtight

Airtight Assemblies that are massive, airtight, and structurally discontinuous do the best job keeping out neighboor's TV noise, or keeping out the bus noise, from your apartment. By contrast, sound absorption is used to reduce the sound buildup inside the same room where the sound is made, and has less impact on the neighbor's noise.

The owner of a sandwich shop wishes to open a new location on an empty site near a shopping center. There are three entrances to the building: one that faces the shopping center to the north, one that faces a public parking area to the south, and a service entrance to the west that leads to the dumpsters and walk-in cooler. Which of the entrances is required to be accessible?

All of the entrances must be accessible Correct. Since the service door serves a specific space, it would also need to be accessible, as well as the other two doors.

The sun's position in the sky along the vertical axis is called its ________.

Altitude

What is an Aquifer thermal

An aquifer thermal energy storage system (ATES) is a type of open-loop geothermal system which stores cold water from winter in the aquifer for use during the summer months as cooling. This flattens the curve seasonally, but not daily.

What is economizer?

An economizer is a mechanical device that reduces the amount of energy used to cool a data center or other buildings. It is integrated into the building's heating, ventilation and air conditioning (HVAC) systems, which is why it is sometimes called a HVAC economizer.

Laminated Veneer Lumber (LVL)

An engineered wood product in which veneers of wood are stacked in parallel and glued under pressure

What is the proper foundation waterproofing detail

Are

What are Floor cellular re¡aceways best for

Best for new construction. The floor slab is poured with a floor cellular raceway.

What are Poke thorugh floor boxes best for

Best for retrofits and renovations because the floor slab is already poured.

An architect is working on the design of a new office facility. The project requires sustainable materials and practices, so the architect is performing a life cycle analysis on the building envelope systems. Which of the following tools provides a life cycle analysis using materials and values specific to the United States?

Building for Environmental and Economic Sustainability (BEES)

Which of the following passive or active solar features will have the most impact on the overall site design?

Building location Correct. The selection of an appropriate site can mitigate a lot of adverse site characteristics and inform the overall design.

An architect is designing a day care facility, and the client is concerned about off-gassing affecting the indoor air quality. Which material, when installed, emits the least amount of dangerous off-gas?

Ceramic tile Correct. Clay is relatively free of toxins and the firing process seals ceramic tiles, preventing them from giving off toxins or absorbing anything.

Which alloless moisture to Pass? Class 1 Vapor Retarder CLass 2 Vapor Retarder Class 2 Vapor Retarder

Class 1 Vapor Retarder

An architect is designing a theater with a control room that will house lighting and acoustical control equipment. WHich type of portable dire extinguisher should the architect specify for this room?

Class C are used on fires involving electrical equipment

In which climate should a row of housing be used for protection against wind?

Cold Row or cluster housing is most appropriate for cold climates where protection from the cold winter winds is a priority

An architect is designing a daycare facility and the client is concerned about off-gassing affecting the indoor air quality. When installed, which material emits the least amount of dangerous off-gassing of volatile organic compounds (VOCs)?

Concrete Correct. Concrete, stone, masonry, ceramic tile, cementitious mortars, and grouts are chemically inert and generally free of emissions. Carpet Incorrect. The adhesives in carpet can contain and emit VOCs.

What is the difference between concrete and cement?

Concrete is a composition of cement and aggregate What is the difference between cement and concrete? Although the terms cement and concrete often are used interchangeably, cement is actually an ingredient of concrete. Concrete is a mixture of aggregates and paste. The aggregates are sand and gravel or crushed stone; the paste is water and portland cement.

What is a Split system?

Condenser outside, evaporator inside

A studio director is currently estimating the shaft space required for a high-rise office building during the schematic design phase. The typical floor plate is 10,000 square feet. How many square feet of space should be set aside for shaft space?

Correct. For high-rise buildings, approximately 4% of the floor area should be set aside for shaft space as a preliminary estimate. This will be refined as the building systems are coordinated with the engineers.

After bidding a project for a new office tower, the design team discovers the project is over budget. To reduce costs, the project architect decides to engage the team in a value engineering exercise. Which of the following strategies can assist in the process of value engineering?

Creating mock-ups of assemblies Correct. Creating mock-ups of systems and assemblies can help open the door to innovative design discussions and solutions. Mock-ups offer the design team a chance to interact with and better understand a system or assembly. By better understanding the item, the design team can then search for cheaper solutions to achieve the same or similar results. Reviewing client-preferred products Incorrect. Architects generally specify products requested by the client. In the process of value engineering, the architect and design team would typically be looking to other sources to find less expensive, comparable items.

Which open-web steel joist type spans the longest? LH DLH K

DLH, span about 150'

An architect is designing a 2-story seasonal home in Colorado. The client is concerned about solar gain throughout the year and how it will affect the interior temperatures. In the northern hemisphere, solar gain is the lowest during which month?

December Correct. The least solar gain occurs when the sun is the lowest in the sky, which occurs on December 21-22, during the winter solstice.

What are accelerators on a concrete mix good for?

Decrease the setting time of a mix, allowing concrete to be placed in winter with reduced risk of frost damage.

An architect is considering adding a photovoltaic panel array to a new office building. The owner has asked that the new facility be flexible and easily adaptable as the business grows. Which aspects of photovoltaic cells will help the facility be adaptable as the business grows?

Decreasing cost Correct. As demand for photovoltaic systems continues to increase, the price has started to decrease, making these systems a more affordable option for renewable energy. Multiple applications Correct. In addition to their standard panel form, photovoltaic panels can be incorporated into multiple different types of applications such as curtain walls and roofing materials, allowing for a variety of aesthetics. Modular construction Correct. Each panel is able to be removed and replaced individually, which allows for flexibility in adding to the system or replacing panels as necessary. Net metering capability Correct. By utilizing a photovoltaic system with net metering, the owner could have the ability to sell any unused energy back to the utility grid. Easy to recycle at end of life Incorrect. While recyclability is good for the environment, very few solar panels are actually recycled due to the high cost associated with recycling them.

An architect is considering adding a photovoltaic panel array to a new office building. The owner has asked that the new facility be flexible and easily adaptable as the business grows. Which aspects of photovoltaic cells will help the facility be adaptable as the business grows? Check the four that apply.

Decreasing cost Correct. As demand for photovoltaic systems continues to increase, the price has started to decrease, making these systems a more affordable option for renewable energy. multiple applications Correct. In addition to their standard panel form, photovoltaic panels can be incorporated into multiple different types of applications such as curtain walls and roofing materials, allowing for a variety of aesthetics. Net metering capability Correct. By utilizing a photovoltaic system with net metering, the owner could have the ability to sell any unused energy back to the utility grid. Consistent power generation Incorrect. Since photovoltaic systems rely on solar energy to generate power, the overall performance of the system can be intermittent as weather conditions (i.e. clouds, darkness) can limit energy production.

An architect is working on a space planning exercise for the owner of an existing commercial building. The owner is interested in studying a variety of alternatives for how the typical floor plan can be divided for different tenant options. Which of the following aspects of the space should be included on the plans to present to the owner?

Demising walls Correct. Demising walls that separate the tenants from each other and from the corridor are critical to show on this type of plan. Square footages Correct. The owner is likely concerned with the sizes of each space, since the amount of money for which office space can be leased is typically based on the square footage of the space. Workstations Correct. It is important to plan for an optimal workstation layout when performing space planning exercises for an office building. Windows Correct. The locations of exterior windows will likely drive the locations of demising walls, and will have an effect on the desirability of each office suite. Exit signs Incorrect. Exit signs, as well as other reflected ceiling plan information, are not typically shown on a space planning diagram.

A new three-story building is being designed in the western United States. The client was hoping to build with masonry to achieve a historic look, but has been convinced to use steel for its structural properties. A steel-framed building can better withstand the effects of an earthquake over a masonry building due to which property?

Ductility Correct. The unique structural property of steel is its high ductility. This refers to the elongation that a material can undergo after it has yielded or exceeded its elastic range. Steel has a long range of ductile ability in which it can be subjected to higher and higher stresses without failure. In good seismic design, structures should yield in order to absorb and dissipate energy. Ductility in seismic design applies to the structure as a whole, as well as the members and connections. Strength Incorrect. Strength measures how much stress can be applied to an element before it deforms permanently or fractures. Strength of a material can refer to either yield strength, ultimate strength, or fracture strength. A material can be strong without being ductile. A steel wire is very strong but will break suddenly.

An architect is coordinating the design of the lighting and electrical receptacle locations for an amphitheater project. The consulting electrical engineer recommends a line voltage system over a low voltage system. Why should the architect take the engineer's recommendation to use a line voltage system?

Easy Integration of systems over large areas Line voltage systems have a lower voltage drop and can carry power for greater distances than low voltage systems.

Which of the following would be the best source for the architect to obtain information to inform their cost estimate in a cost-effective manner?

Estimating manual Published cost-estimating manuals provide square footage costs based on type of building, quality and complexity of construction, and geographic location. They can be a good source for quick and simple estimates. Note: Hiring a cost-estimating consultant can be expensive. Additionally, since these are private discussions between the architect and owner, hiring a cost consultant would not be appropriate.

An architect is working on the restoration of a historic house. The original divided-light windows are in working order, but some need to be reglazed with glazier's points in their rabbeted wood frames to match the existing glazing. What type of glazing system is the most appropriate for this application?

Face glazing Correct. Face glazing seals the individual glass panes in rabbeted frames with a beveled bead of either putty or glazing compound. The glass is held in place with glazier's points until the face putty or compound hardens. Face glazing is typically how older wood windows were set.

What is an Air-to-air system?

Fan over condenser and evaporator

What is an Air-to-water system?

Fan over condenser, pumped water over evaporator

An architect is designing a new office building, and has determined it should be Type IIB construction to fulfill all program and code requirements. The architect's design could include which of the following systems?

Fiberglass acoustic panels Correct. IIB construction requires noncombustible materials. Interior full-height glass walls Correct. IIB construction requires noncombustible materials. Precast concrete panels Correct. IIB construction requires noncombustible materials.

An architect has been commissioned to design a mid-rise office building. The architect would like to increase the floor area of the building without changing construction types. Which of the following will allow the increase?

Firewall(s) Correct. Adding a firewall that extends continuously from the foundation through the roof would allow the area under the roof to be increased without changing construction types. Fire partition(s) Incorrect. A fire partition is a vertical assembly of materials that are typically used as an interior wall. This element can't be used to separate the building or increase the fire area.

Rotation Free and translation free Connection?

Flag Pole

The architect needs to specify a paint finish for an existing painted exterior brick wall with blemishes. Which finish should the architect specify in order to minimize attention to the blemishes?

Flat Correct. Because of the low sheen of flat paint, blemishes are easier to disguise. Flat finishes are harder to clean, however, because they are somewhat porous.

What is the difference between Footcandles & lumen se?

Footcandles Often used by architects represent the amount of light that needs to be transmitted to a surface depending on the space's usage. This is often referred to as SF over the X & Y of the point source. Lumen is the intensity of the light source in order to meet the proper demand.

A new client has come to an architecture firm by way of an existing client and will be developing a 50-million-dollar apartment community. They would like one of the buildings to have 65,000 square feet of retail with apartments above. The owner's representative requests that the building uses wood-framed construction. The senior project architect on the team suggests using a 1-hour fire-rated building frame with a 3-hour horizontal separation between the retail space and apartments. What is the maximum number of stories of apartments above the retail?

Four stories Correct. The 3-hour separation would allow the retail and apartments to function as two separate buildings for the purposes of determining area and number of stories. Wood-framed construction is Type V, but having a 1-hour fire-rated structural frame makes the construction Type VA. Apartments are classified as R-2 occupancy. R-2 occupancies are required to be sprinklered. Per Table 504.4 in the IBC, the maximum allowed number of stories for an R-2 occupancy of Type VA construction, with a sprinkler system or 13R sprinkler system, is 4 stories.

An architect is working on a flexible performance space and is selecting the lighting fixtures. What type of lens should the light fixture have in order to provide a concentrated, focused light?

Fresnel Correct. Fresnel lenses have concentric prismatic grooves to concentrate and focus light. Double concave Incorrect. A double concave lens causes light waves to diverge or scatter, not focus. Light houses uses Fresner Lenses

Shaft

Functions quite similarly to a chase, but it is intended to be fire rated

What is a fundamental period?

Fundamental periods are the behavior of a building or object under seismic loads. All structures and objects have a fundamental period.

Based upon the lack of existing site utilities and the rural nature of the proposed site, which type of mechanical system is most appropriate for this project?

Ground source heat pump Correct. A ground source heat pump utilizes the ambient temperature of the ground to modulate the air used to heat and cool a building. It is much more energy efficient than any other traditional vac system, and it would cater to the need to have a minimal energy usage requirement on the given rural site.

During the design development stage of a regional airport project, an architect notices a spatial conflict between a return air duct and several light fixtures serving the main entrance. How should the architect determine the best course of action?

Have the HVAC consultant redesign the duct Correct. The most efficient and design-conscious approach is to adjust the return air. This could result in the rerouting or resizing of the duct, or shifting to a return air plenum. Since return air is not hampered by corners or turns, it simply needs negative pressure to reach its destination. As the light fixture serves a lobby space, it is likely integral to the design, and would involve more work to adjust. Have the lighting consultant redesign the lighting Incorrect. As the lighting serves the main entrances, the location of the light fixture should take priority.

The client wants an efficcient heating system to help minimize cost. Which of the following heat sources could the architect recommend?

Heat Pump A heat pump is used to move heat, not just convert energy to heat. Therefore, its annual fuel utilization efficiency (AFUE) is greater than 1. A coefficient of performance (COP) is the ratio of the rate of heat delivered to the rate of energy input. Heat pumps have COPs that are greater than 1 and sometimes 2 or 3. This means the output is upwards of three times the energy input.

The client wants an efficient heating system to help minimize cost. Which of the following heat sources could the architect recommend?

Heat pump Correct. A heat pump is used to move heat, not just convert energy to heat. Therefore, its annual fuel utilization efficiency (AFUE) is greater than 1. A coefficient of performance (COP) is the ratio of the rate of heat delivered to the rate of energy input. Heat pumps have COPs that are greater than 1 and sometimes 2 or 3. This means the output is upwards of three times the energy input. Boiler Incorrect. A boiler converts energy to heat with minimal loss in efficiency. A standard efficiency furnace provides 90% annual fuel utilization efficiency (AFUE). In this type of furnace, 10% of the energy produced by natural gas is expelled as exhaust or latent heat. This is not the best answer, as it is .9 efficient.

A large airport project requires simultaneous heating and cooling for various program spaces across a terminal. Which type of HVAC system would be best for the project?

Heat recovery variable refrigerant flow (VRF) system Correct. The heat recovery system can provide both heating and cooling at the same time. This is ideal if the building has several rooms or separate programmatic spaces. Heat pump variable refrigerant flow (VRF) system Incorrect. A heat pump VRF system is similar to a heat recovery VRF system as it can provide either heating or cooling to a building, but it cannot do both at the same time.

A large airport project requires simultaneous heating and cooling for various program spaces across a terminal. Which type of HVAC system would be best for the project?

Heat recovery variable refrigerant flow (VRF) system Correct. The heat recovery system can provide both heating and cooling at the same time. This is ideal if the building has several rooms or separate programmatic spaces. Variable air volume (VAV) system Incorrect. A variable air volume system can provide either heating or cooling to a building, but it cannot do both at the same time.

An architect is classifying a large building that is used for the manufacture and storage of consumer fireworks. How should the building be classified?

High-hazard Correct. The high-hazard occupancy group includes buildings for the manufacturing and storage of hazardous materials, such as consumer fireworks.

What is a Condenser?

High-pressure hot refrigerant

An Architect is selecting an elevator for a three-story residential building that serves 12 apartments. The initial cost is the developer's primary consideration. Which one of the following systems should the architect select?

Holeless hydraulic elevator This elevator can serve lift heights up to 40 feet so it will work for a three-story building. WHile a holeless hydraulic elevator will have higher operating costs, it will have the lowest initial cost., which is what the developer desires.

A seven-story structure is being planned. The structural system must minimize on-site construction time, and the floor thickness should be as thin as possible. Which structural system should the architect select?

Hollow core slab Correct. A hollow core slab minimizes site construction time and allows for thin floor thickness. Reinforced two-way flat plate Incorrect. Two-way flat plates are thin, but do not minimize construction time.

An architect is developing the code review documents for a new medical facility. While performing the code review, the architect notices that certain types of medical facilities are classified as an Institutional occupancy and will require additional considerations. Which of the following medical facilities would be considered an Institutional occupancy?

Hospitals Correct. A hospital is classified as an I-2 Institutional occupancy. Assisted living facility Correct. An assisted living facility is an I-1 Institutional occupancy. Rehabilitation facility Correct. A rehabilitation facility is classified as an I-1 Institutional occupancy. Ambulatory care facility Incorrect. An ambulatory care facility is classified as a Business (B) occupancy. Outpatient clinics Incorrect. An outpatient clinic is classified as a Business (B) occupancy.

An electrical engineer wants to look up the minimum required number of footcandles for interior artificial lighting. Where should they look?

IBC (International Building Code) - Chapter 12 Correct. IBC Chapter 12 addresses the Interior Environment. Section 1204.3 states the minimum required footcandles for interior artificial lighting.

Peak demand for cooling often coincides with peak demand for electricity. This results in a spike in electricity use when electricity is most costly. Which type of cooling flattens this curve?

Ice storage Correct. An ice storage cooling system uses energy overnight, when it is less expensive, to freeze ice. The ice is then used during the day to cool the building. Evaporative chiller Incorrect. An evaporative chiller uses the latent heat of vaporization to cool air, but this does not flatten the demand curve like an ice storage system.

An architect is designing a new Prairie-style house with three-foot roof eaves on a 50-foot-wide lot. The zoning ordinance requires five-foot side yard setbacks and allows the roof eaves to project no more than two feet into the setbacks. What would be the maximum allowable width of the house per the zoning ordinance?

If the house has three-foot wide eaves and the zoning ordinance allows the eaves to project up to two feet into the side yard setback, then the exterior walls of the house would need to be located one foot to the inside of each side yard setback.50' wide lot - 10' (two five-foot side yard setbacks) - 2' (two additional one-foot setbacks due to eaves) = 38' maximum house width.

An architect is designing a sign that must be legible to pedestrians from a distance of 60 feet away. The architect is following a guide that states that the letters are to be 3 inches high for the first 21 feet of viewing distance and an additional 1/8" higher for every foot of viewing distance beyond that. What is the minimum height for the letters on the sign? Round to the nearest inch.

Incorrect - the answer is 8 Find how many inches need to be added to the base of 3 inches in order to be readable at a distance of 60 feet.60 feet - 21 feet = 39 feet. 39 x 0.125" = 4.875". Adding 4.875" to 3" = 7.875". Round up to 8".

SHould we vent or Insulate the crawlspace?

Insulate

An architect is working on a project in an unfamiliar location. Where should the architect look to determine the climate zone of the project site?

International Energy Conservation Code (IECC) Correct. Climate data for building science is found in the International Energy Conservation Code. These maps are used to determine the climate zone for designing building envelopes and mechanical systems in buildings.

What is Ultraviolet radiation used for?

Is used for disinfecting the water supply.

An architect is designing a passive solar building that will take advantage of an adjacent south-facing slope. The slope will have a specially designed collector where the air will be warmed by the sun and then rise to serve as the heat for the building. Cooler air will simultaneously be pulled from a chamber below the floor, creating a convection loop. How is this type of passive solar system identified?

Isolated gain Correct. Isolated gain systems collect and store solar radiation away from the space to be heated, as described.

An architect is designing a new office building near a freeway. The owner requests large amounts of glazing to optimize natural lighting in the interior of the building. Sounds from the freeway are a concern. What should the architect specify for the glass?

Laminated Correct. Laminated glass is effective at reducing sound transmission.

What is a Direct expansion (DX) unit?

Like a window unit; with all cooling components including refrigeration machine and fan in one box.

An architect is working on a new restaurant buildout where manual dampers will be required in the ductwork system. Where in the ductwork, for RTU-1 (Rooftop Unit-1) and RTU-2 (Rooftop Unit-2), should the manual dampers be located? Using the image below, drag the designation into the correct locations.

Manual dampers are used to manually control the airflow through the ductwork. For RTU-1, the dampers are placed close to the main 32"x16" trunk, in the 16"'x12" lines in order for each line to be controlled separately. For RTU-2, the dampers are placed in each of the 14"x10" lines in order for them to be controlled separately.

What is the difference between pre-stressed, pre-tensioned, and post-tensioned concrete?

Most precast, prestressed concrete is actually pre-tensioned-the steel is pulled before the concrete is poured. Post-tensioned concrete means that the concrete is poured and then the tension is applied-but it is still stressed before the loads are applied so it is still prestressed. Pre-stressed is squeezing concrete together. Pre-tensioned (such as precast concrete) and post-tensioned (such as cast-in-place concrete) are categories of Pre-stressed concrete.

An architect is designing a new school and has performed a building code review. During the code review, it was determined that the building will need a sprinkler system. Which of the following is the appropriate standard for sprinkler system design?

NFPA 13 Correct. NFPA 13 is the appropriate standard for sprinkler system design in schools.

A flat, open site is being evaluated for a new hotel, and a windbreak will be incorporated to reduce wind. What is true regarding air flow around windbreaks?

Negative Pressure at sides There is negative pressure at the sides and leeward surfaces of a windbreak. (Note: there is an increase in pressure at the windward face of the windbreak)

You are designing a high school football stadium with an elevated drum major platform at 24" high. Do you need a railing?

No railing required for heights below 30 inches.

Is frost level part of a geotechincal report?

No, The frost level is not part of a soil report, but is a predetermined depth of frozen subsoil for a particular region of the country.

What is the N.R.C.?

Noise Reduction Coefficient (NRC) (How much quieter a product can make your space.) Like a sponge absorbs water, an acoustic product absorbs sound and the NRC tells us just how much sound those products can soak up. NRC is measured on a scale that ranges from 0 to 1. An NRC of 0 means that the product absorbs no sound. For example, an NRC rating of . 75 means 75% of the sound energy coming in contact with that specific material is absorbed. Simply put, the sound is not reflected back into the room to create noise. A material with an NRC of 0.75 would also be considered 25% reflective.

An architect is calculating the occupancy load for a building. Which of the following requirements are dependent on the occupancy load?

Number of exits Correct. Occupancy load is used to determine number of required exits. The direction of door swings Correct. Occupancy load is used to determine door swings. Width of egress stairs Maximum building height and area Incorrect. Zoning regulations and type of construction, in conjunction with occupancy use group, determines building height and area limitations.

A structural engineer is designing for the loads of several multi-story buildings that have different uses. For which of the following buildings should the engineer design for a partition load?

Office buildings Correct. In buildings where partition locations are likely to change, such as office buildings, provisions for a partition load of 15 psf shall be made, if the live load is less than 80 psf. See International Building Code (IBC, 2018) Section 1607.5. Apartment buildings Incorrect. Room sizes in apartment buildings are standard and therefore, partitions are unlikely to change. See International Building Code (IBC, 2018) Section 1607.5.

What is one BTU in tons?

One ton of capacity is equivalent to 12,000 BTU per hour or 3.5 kW

Types of one way slab systems?

One way slab One way slab - Solid slab One way slab - Ribbed Slab Wide module slab Tuypically use on a rectangular base. spans > 30'

A developer is planning on building a new riverfront restaurant on a downtown site that is zoned for business use. Because the site is also located in a flood zone, there are additional flood protection standards for the building that would need to be addressed. What type of zoning regulation is described in this scenario?

Overlay district Correct. An overlay district is a special district superimposed over parts of one or more underlying standard use zoning districts so that additional special use standards can be addressed. In this case, the flood zone district is an overlay on the standard business district, and would require additional flood protections for properties in that zone. Conditional use permit Incorrect. A conditional use permit allows for a zoning use that is not normally allowed in a particular district, but is compatible with the character of the zoning district.

There are various types of forces exerted upon a building that affect the building's structure. Natural forces such as an earthquake can cause a building to sway back and forth. This motion results in additional forces that are identified as which of the following?

P-delta effect Correct. The secondary effect of force due to the sway of a building is called the P-delta effect. Moment Incorrect. Moment is the application of a force that causes bending, and noCCCCCCCCCCCCCCCCCCCt a secondary effect of lateral forces.

What's the formula for pressure?

P=0.433xH

An architect has been hired to design an 8-story office building. The client wants the structure to be environmentally responsible and resource-efficient. The architect is preparing a specification that calls for Green Seal Standard GS-11. What is the architect specifying?

Paint Correct. Green Seal Standard GS-11 refers to a standard set of environmental, health, and performance requirements for sustainable paints and coatings.

An architect has been hired to design an 8-story office building. The client wants the structure to be environmentally responsible and resource-efficient. The architect is preparing a specification that calls for Green Seal Standard GS-11. What is the architect specifying?

Paint Correct. Green Seal Standard GS-11 refers to a standard set of environmental, health, and performance requirements for sustainable paints and coatings.

Standpipes

Pipes that run the height of a building and provide water outlets at each floor to which fire fighting hoses can be connected

How do we best reduce the build-up of low-frequency sound in a room ( For instance, rumble from mechanical equipment)?

Position sound-absorbing material near the corners and edges of walls.

An architect is specifying a finish for exposed metal. The architect wants to select the ost sustainable finish. Which one of the following finishes should the architect specify?

Powder coating When metal coatings are required, powder-coated fabrications can be specified as a solvent-free application. The powder overspray can also be reclaimed for reuse.

An Architect is specifying a fire suppression system for a small museum building that will house sensitive and expensive artifacts. Which sprinkler system should he specified?

Preaction System A preaction system must undergo a two-step process before the sprinkler releases the water. This system is designed to protect spaces with sensitive and expensive materials.

What is the compressor ?

Pump that circulates refrigerant

An architect is working on a low-rise manufacturing building with a large footprint in a greenfield development. One of the client's sustainability goals is to use stormwater to reduce potable water consumption on the site. To meet the client's goal, what structure should the architect consider implementing when developing the site plan?

Rainwater collection system Correct. There is a diverse array of rainwater harvesting strategies which can be adapted to utilize diverted water within and around the building. For example, harvested rainwater can be used to irrigate plants and flush toilets. Underground detention basin Incorrect. A detention basin is utilized as a flood control measure and will not provide opportunities to use the diverted water.

When an economizer turns on, another part of the mechanical system turns off, saving energy. Which portion of the mechanical system is turned off by the start of the economizer?

Refrigerant cycle Correct. An economizer cycle in an HVAC system can provide cooling when the outdoor temperature is cool enough to provide it, allowing the refrigerant cycle to turn off, saving energy and money.

What is a Mini Split System?

Refrigerant flows through units in rooms under high pressure for heating and low pressure for cooling; can heat and cool different rooms simultaneously

An architect has been hired to perform a programming analysis for a new preschool. The architect's firm specializes in educational facility design but has not recently completed a preschool. What is the first step the architect should take in the programming process?

Research the project type Correct. This is the first step in a programming exercise. It is important to study precedents before proceeding further with any analysis, especially since the firm has not completed a preschool design in the recent past. Identify programming strategies Incorrect. This step can only be successfully done after the architect has researched the project type, established goals, gathered data, and analyzed the data.

An architect has been hired to perform a programming analysis for a new preschool. THe architect's firm specialized in educational facility design, but has not recentlyy completed a preschool. What is the first step the architect should take in the programming process?

Research the project type This is the first step in a programming exercise. It is important to study precedents before proceeding further with any analysis, especially since the firm has not completed a preschool design in the recent past. Identify programming strategies Can only be successfully done after the architect has researched the project type, established goals, gathered data, and analyzed the data.

The owner provides a topographic survey of the greenfield site of a new shopping mall. The architect noted the site has several steep vegetated hills. What site strategies should be put in place to mitigate erosion?

Revegetate and mulch exposed areas Correct. Added vegetation and mulch help to hold disturbed soil in place and limit erosion from runoff. Protect and retain existing vegetation Correct. Existing vegetation is already established on the hillside and helps prevent erosion from rainwater. Design development to fit the site and the terrain Correct. Situating the building on an area of the site with flat terrain will limit disturbance of existing soils, which leads to a greater chance of erosion. Grade the site by filling in channels Incorrect. Filling in channels will lower natural flow capacity, causing them to receive larger amounts of water in shorter periods of time, will lead to erosion.

Which type of soil is more stable to build on? Clay or Sand

Sand Clay behaves unpredictably when it gets wet. It swells Of course most soil boring reports detail a mix of sand and clay (and silt and gravel). It is then the proportion of clay that will determine stable soil.

Wood chart summary

See chart

What are helical piles?

Sometimes referred to as scre-piles. These piles screw into the soil like nail screws into wood. This means there is no soil displacement, making it suitable for installations where minimizing impact is the main criterion.

An architect is detailing a wall assembly for a multi-family residential building. Which of the following wall types will provide the greatest thermal and acoustic insulation?

Staggered stud with insulation Correct. A staggered stud wall with insulation would provide the greatest thermal and acoustic insulation, because staggering the studs prevents thermal bridges and reduces the transmission of acoustic vibrations. 8" CMU, painted Incorrect. An 8-inch CMU wall would provide some level of acoustic insulation, but would not provide adequate thermal insulation. Double-wythe CMU wall Incorrect. A double-wythe CMU wall would provide some thermal and acoustic insulation, but not as much as the staggered stud wall.

An architect is designing an office building and the top floor of the building will be more than 30 feet above the lowest level of fire department vehicle access. Which fire protection system is required by the International Building Code for this condition?

Standpipes Correct. Buildings where the floor level of the highest story is located more than 30 feet above the lowest level of fire department access shall be equipped with a standpipe. Annunciator panel Incorrect. An annunciator panel is a unit which contains displays and equipment to provide information on the condition of a building. It does not relate to water supply or range of extinguishing potential, which is what the question is asking.

What metal has the lower embodied energy? Aluminum or Steel

Steel (90%steel is recycled) 30% aluminum is recycled.

How do steel columns distribute their load to the concrete foundation?

Steel baseplates

An architect is calculating the allowable area of a building in accordance with the International Building Code. Which areas are included in the area calculation?

Storage Rooms Lobby Vestibule Per IBC the area of a building is defined as "the are included within surrounding exterior walls (or exterior walls and fire walls) exclusive of vent shafts and courts. Areas of the building not provided with a surrounding wall shall be included in the building if such areas are included within the horizontal projection of the roof or floor above. Note the following are not included in the allowable building area calculation: Mezzanine Basement Exterior walls Sub-basement

The owner of a private art collection is looking at two potential building sites for his next gallery. The owner's main desire is to display the artwork in a naturally lit space. Which of the following site analysis diagrams would help the architect choose a site that fits the owner's main desire? Check the three that apply.

Sun path diagram Correct. This diagram would help in choosing the site that can best provide daylight at specific hours of operation and help in determining any necessary shading. Vegetation diagram Correct. This diagram can show where existing mature planting is located that may hinder daylight penetration into the space and potential site situation. Building height diagram Correct. A building height diagram can inform the owner of the surrounding building heights and potential for casting unwanted shadows on the site, limiting the potential for daylight.

An architect is designing a shelter at a remote site in an arid climate where water is scarce. Clay found on the site will be used to create bricks for the primary construction material. The bricks will be bonded with a cement mortar. What type of admixture should the architect specify to reduce the amount of water required in the mortar mix?

Superplasticizers Correct. Superplasticizers improve the workability of a concrete or mortar mix at a given water-cement ratio. They can also be used to reduce the amount of mixing water needed for a desired workability. Surfactants Incorrect. Surfactants reduce the surface tension of the water in a concrete or mortar mix. This helps the cement particles and other additives in the mix to emulsify and disperse evenly.

An architect has just entered into a contract with a new client. The architect is explaining to their team which design alternative studies are included in the scope of basic services and which should be performed as an additional service. Which of the following design alternative studies are covered under the scope of the architect's basic services, per the AIA Standard Form of Agreement between Owner and Architect?

Sustainable design alternatives Correct. Per AIA B101-2017 Article 3.2.5.1, the architect shall consider sustainable design alternatives as part of the basic services. Inclusion of design alternates in specifications Correct. Per AIA B101-2017 Article 4.2.1.6, the architect shall include design alternatives for bidding purposes in the contract documents, which include the specifications. Alternative materials Correct. Per AIA B101-2017 Article 3.2.5.2, the architect shall consider alternative materials as part of the basic services. Alternate building systems Correct. Per AIA B101-2017 Article 3.2.5.2, the architect shall consider alternative building systems as part of the basic services. Multiple preliminary designs Incorrect. Per AIA B101-2017, this is listed as an additional service.

When is tempere glass required? When is laminated glass required?

Tempered glass: 5x stronger than untempered annealed glass. Required at storefront door (or glass panel adjacent to a door) where the glass extends to 18" above finished floor. (Stronger to impact) Laminated glass: Stronger still and when it breaks, shards generally stay glued to the laminate. Laminated glass is required for skylights, and hurricane-prone regions, as well as security-sensitive rooms like prisons, mental hospitals, or casino cashiers. (More expensive)

What is the bending moment of a beam?

The Bending moment is the breaking point of a beam due to its long span and weight. Moment is a function of L^2

absorption coefficient

The absorption coefficient determines how far into a material light of a particular wavelength can penetrate before it is absorbed. In a material with a low absorption coefficient, light is only poorly absorbed, and if the material is thin enough, it will appear transparent to that wavelength. An absorption coefficient of 1 means that all acoustic energy striking the surface will be absorbed and none reflected. A coefficient of 0 means that all the energy is reflected.

An architect is performing the code analysis for a 100-unit apartment building. The owner plans to acquire low income housing tax credits for the building. How many of these units are required to be Type A accessible units?

The answer is 2 Correct. 1107.6.2.2.1 of the 2018 International Building Code (IBC) requires that 2% of the units shall be Type A. 100 units x 0.02 = 2 Type A units

An architect is designing a new residential building for a client who prefers to meet the minimum code requirements and nothing more. Each residential unit will have a master bedroom that is 500 square feet and a second bedroom that is 250 square feet. What is the minimum area of glazing required for the two bedrooms combined?

The answer is 8640 Correct. IBC (2018) 1204.2 requires a minimum net glazed area of 8% of the floor area. The total area of the two rooms is 750 sq. ft., and 8% of that would be 60 sq. ft. (8,640 sq. in.). multiply the area value by (12x12) 144 to get SQ inches.

What is the fundamental period?

The behavior of a building or object under seismic loads. All structures and objects have a fundamental period.

What s an exit discharge?

The exit discharge is the portion of the means of egress system that is between the termination of an exit and the public way.

What is the maximum size of a mezzanine?

The maximum area of a mezzanine is 1/3 of the size of the room in which it is located.

An architect has chosen to use block planning to create a preliminary program layout. What is the primary advantage that block planning has over bubble diagramming?

The result resembles a floor plan Correct. A block plan avoids curvilinear and non-rectangular solutions, so the result more closely resembles a floor plan Circulation is included Incorrect. Circulation is typically included in both bubble diagrams and block plans.

An architect is in the preliminary stage of design for a new high school building. The mechanical designer has suggested using a central HVAC all-water system. Which of the following is true regarding a central HVAC all-water system?

The system offers no humidification Correct. One major disadvantage to an all-water system is that it provides no humidification. Chiller efficiency is lower than it would be for individual heat pumps Incorrect. The chiller efficiency is higher than it would be for individual heat pumps.

What is convection?

The transfer of heat by the movement of a fluid

An architect is performing a code review for a new school building. During the review, the architect reviews the International Energy Conservation Code in addition to the International Building Code. Which of the following items are controlled by the International Energy Conservation Code (IECC)? Check the four that apply.

Total building performance requirements Correct. The IECC denotes the requirements for total building performance. Lighting controls Correct. The IECC specifies lighting control requirements to maximize energy efficiency. Wall insulation requirements Correct. The IECC specifies the wall insulation requirements. Glazing thickness Incorrect. While the IECC specifies thermal performance criteria for glazing, the overall construction of the glazing assembly is controlled by the IBC.

What type of concreted do you use for Pre-cast concrete?

Type III - High early-strength concrete. Useful for precast concrete, concrete block, tilt-up concrete, and anywhere else where we want to strip the formwork soon after pouring. Also used for cold-weather construction. Type I - Normal Type IA - Normal, Air entrained Type II - Moderate resistance to sulfates. Type IIA - Moderate sulfate resistance, air entraining etc.

WHen do you use a two way lab?

Typically when the area being supported is square The Building Structural Standard defines a two-way slab system as a concrete slab system in which two rebars are arranged in two directions regardless of the presence or absence of a beam that transmits a load to a column'.

An architect is planning a new auto mechanic shop. The shop will have two repair bays, a storage area, and a small office. The building will be 1,000 square feet. The building is to be categorized as the least possible fire-resistive construction type. Which construction type is both appropriate and minimally fire-resistive?

V-B Correct. Type V-B construction is unprotected wood frame. This type has no fire rating requirement, making it the most combustible. It is appropriate for the auto shop because there are multiple occupancy types and the square footage is low enough to fit within the building area requirements of Type V-B construction. V-A Incorrect. Type V-A construction is protected wood frame. This requires the structural framing, bearing walls, floor, ceiling, and roof to have a one-hour rating. III-B Incorrect. Type III-B construction is unprotected combustible construction with brick or block walls. The exterior walls need to have a two-hour rating.

A set of plans is designed for a cold climate. A builder wishes to build the same structure in a warm climate. Some of the details will need to be modified to account for the change in the climate. What building material should the architect relocate within the wall section?

Vapor retarder/barrier Correct. Vapor retarders (or vapor barriers as they are sometimes called) are placed in the wall on the warm side of the insulation. This means it goes on the inside, in a cold climate, and on the outside, in a warm climate.

The owner of a proposed three-story office building would like their architect to design the building so that it will receive LEED Gold certification and meet their specific energy usage goals. While the architect cannot guarantee this, they explain to the owner that if the building is built in accordance with their drawings and specifications, it will be capable of obtaining the desired certification and meeting the energy usage goals. Which of the following factors that occur after design completion could affect the achievement of these sustainability goals?

Value engineering Correct. Value engineering by the owner or contractor to reduce costs during construction can have an impact on overall building and systems performance and certification. The architect should notify the owner and contractor of performance or certification issues that could be a result of proposed changes to the drawings and specifications. Certification approval Correct. LEED certification is not guaranteed. Various elements in the final construction and submitted documentation could affect whether certification will be awarded, or what the level of certification will be. Building systems operation and maintenance Correct. When energy usage does not meet specific goals, it is often caused by the owner not operating the building systems as they were designed to be run. Contractor documentation Correct. The contractor must accurately compile the required documentation during construction that will be submitted for LEED certification. If the contractor fails to provide this information, the building may not be certified. Permit approval Incorrect. Receiving a building permit should have no impact on the building meeting sustainability goals, unless the authority having jurisdiction (AHJ) requires revisions to the drawings that will impact those goals. In that case, the architect can try to make revisions that will still be in line with the project goals if possible.

When are seep holes and drainage pipes not required on a retaining wall?

When the retaining wall is 2'-0" or less.

What is a moment connection? (Rotation fixed and translation Fixed)

When you engage the flange and wedge of a system.

Whn do you use a 3 way switch?

When you enter the room from two different points and need to control the light.

Cooling Tower

a cooling system used in industry to cool hot water (by partial evaporation) before reusing it as a coolant. A machine that removes heat from water through evaporation. Warm water is sprayed into the top of the tower, as a fan pulls cool air up from the bottom, across the water, the heat transfers to the air, an is discharged at the top of the tower.

What is a chiller?

a refrigeration system designed to cool liquid

variable refrigerant flow system (VRF)

a system that uses a single compressor and condenser unit located outdoors, connected to multiple evaporators located in different zones of the building. Ideal for office buildings, hotels, and schools. A type of heating ventilating, and/or air-conditioning (HVAC) system which utilizes liquid refrigerant to heat or cool.

What is an EPDM Roof?

an extremely durable synthetic rubber roofing membrane that is used worldwide. EPDM is available in both black and white and is sold in a variety of widths and thicknesses. (Single Ply Roof)

A roof that controls heat gain from the sun is

high Albedo and High emissivity ALbedo is another word for solar reflective Emissivity is the ability to radiate heat

Laminated Strand Lumber (LSL)

made with long strands of wood up to 12 inches in length. Strands are bonded with a resin in a steam-pressing process. typically sued for short strand beams and columns.

Wood I-Joists

manufactured wood I-shaped members, used for framing fr roofs and floors. Made of solid lumber, veneered, strand lumber, plywood or OSB. Can span further than trusses and are lighter. More dimensionally stable and available in longer lengths.

Parallel Strand Lumber (PSL)

structural composite lumber made of wood shreds oriented parallel to the long axis of each piece and bonded together with adhesive

An architect is performing the code analysis for a 100-unit apartment building. The owner plans to acquire low income housing tax credits for the building. How many of these units are required to be Type A accessible units?

the answer is 2 Correct. 1107.6.2.2.1 of the 2018 International Building Code (IBC) requires that 2% of the units shall be Type A. 100 units x 0.02 = 2 Type A units

Variable Air Volume (VAV)

type of HVAC system that varies the volume of conditioned air delivered to room through diffusers in each zone under the control of zone thermostats Somewhat limited in its ability to compensate for extremes for simultaneous heating and cooling demands in a building

What is a shear (pin) connection? (Rotation free and translation fixed)

when you engage only the flange of a system.

cross laminated timber (CLT)

wood product manufactured by layering dimension lumber at right angles to form a thick structural panel. resists axial, bending, & racking loads and provides for lateral loads. very strong. no jobsite waste

When do you have to provide a mid landing and what size does it have to be on a ADA ramp?

Any ramo run can only provide 30" of vertical travel before a landing is required. The landing must be 60" (5'0") minimum in length.

You're designing a renovation of a post-tensioned concrete building. Why is it difficult to simply drill a hole in the floor to win a new vertical pipe?

You could inadvertently core though one of the post-tensioning cables. It's not that unlikely as you might think-there are lots of hidden cables to accidentally cut. We have slab-penetrating sonar tools to find the cables so you wont compromise the entire structure with your hole.

In order to gather information about community concerns regarding a new proposed senior living community, an architect has decided to host a design charrette. The project is privately funded and will use design-build contracts. Throughout the charrette, the architect receives a lot of feedback; some is actionable and some is not. Which of the following responses should the architect consider as relevant regarding the proposed project?

"I'm worried about using my wheelchair to get from my car to the building." Correct. This concern addresses site accessibility. The architect should consider accessibility along the entire path of travel, not just inside the building. "How are we going to address issues of aging in place?" Correct. "Aging in place" is a concept which addresses the ability of a building to accommodate and plan for different levels of accessibility as an occupant ages. This concern should be considered in the design of the new senior living facility. "Will the building be visible from Main Street?" Correct. The architect should consider how the building sits in its surrounding context, including from surrounding thoroughfares. If visibility is a community concern, it's important to address it during the design process to avoid potential backlash from the community in the future.

An architect is working with a contractor to develop a new mixed-use development. As the project design progresses, the contractor provides intermittent constructability reviews to try to keep the project team informed on the cost implications of their decisions. Which of the following are examples of design solutions that increase ease of constructability?

-Provide slotted connections at interior bearing walls -Utilize coursing dimensions at masonry walls -Incorporate prefabricated components -Provide a 1/2" shim space at all exterior glazing By providing a shim space around the window frame, slight adjustments can be made in the field to account for any imperfections in the adjacent construction on site. Note: While scaffolding is required for certain conditions, it is recommended to provide a solid bearing for workers to stand on instead of a scaffolding system.

What is shotcrete?

- lightweight concrete applied by pushing it with compressed air (spraying) - Used for concrete to be placed in difficult locations, thin sections, and large areas. - Advantages: Placing concrete in freeform shapes, repairing structures, thin linings

What are the optimal angles for an active solar panel indifferent location?

-Optimized year-round = Latitude degrees -Optimized for winter only = Lattitude + 15 degrees -Optimized for summer only = Latitude - 15 degrees

An architect is preparing the design and detailing for a below grade basements for a new custom home. The home sits on a site with a variable water table where moisture intrusion and hydrostatic pressure are of concern. Which are the most effective methods for creating a waterproof basement?

-Placing a course of gravel beneAth the foundation pour The gravel course will provide a porous surface for water to drain into the ground below as opposed to soil, which may gather water and cause the capillary or increased hydrostatic pressure on the foundation slab. -Placing a moisture barrier before pouring the foundation Placing a moisture barrier below the foundation will aid in reducing water and moisture penetration up into the the slab. -Placing perforated drain pipe around the foundation, above the concrete footing. Placing the pipe around the foundation will allow water to flow amaya from the foundation to keep the basement dry. Note: Applying damp proofing around the exterior foundation wall is incorrect. Damp proofing is a layer applied during the foundation process, intended to keep soil moisture out of the building. It is not an effective Lon-term solution, as it is unable to seal large cracks in the concrete and may be subjected to damage during backfill operations.

How do the different cuts of wood look and which one is the least expensive?

-Plain Sliced -Quarter Sliced (Most expensive due to more waste) Better looking -Rotary Sliced (Least expensive)

An architect is designing a super-insulated, energy-efficient home in Alaska that utilizes winter passive solar heating through triple-pane windows and implements shading strategies to eliminate unwanted heat gain in the spring, summer, and fall. Which of the following is the best option for the solar heat gain coefficient of the insulated glass units at the south-facing windows?

0.55 Correct. The extremely cold climate of Alaska has a long and demanding heating season and typically no summer cooling demand. The solar heat gain coefficient should be the highest available for triple-pane units, which is approximately 0.55. This allows the window unit to fully utilize winter passive solar heating. The solar heat gain coefficient can only be a value between 0 and 1.

How fast does sound travel?

1 foot per millisecond

An architect is designing a concert hall for small theatrical presentations such as musicals and plays. What is the recommended reverberation time for the concert hall for optimal acoustic quality?

1.1 seconds Correct. 0.8 - 1.9 seconds of reverberation time is suitable for a small theater, a lecture or conference room, or other fairly intimate space. 0.5 seconds Incorrect. This reverberation time would be too short and is best used in a recording or broadcasting studio. 2.4 seconds Incorrect. This reverberation time would be too long and would be better suited for a cathedral.

An architect is designing a concert hall for small theatrical presentations such as musicals and plays. What is the recommended reverberation time for the concert hall for optimal acoustic quality?

1.1 seconds Correct. 0.8 - 1.9 seconds of reverberation time is suitable for a small theater, a lecture or conference room, or other fairly intimate space. 0.5 seconds Incorrect. This reverberation time would too short and is best used in a recording or broadcasting studio.

The architect and owner have decided to use a hydraulic elevator in a 4-story office building. After consulting with the manufacturer, the architect is told that the elevator will need a 10'x12' machine room directly adjacent to the elevator. Which of the following processes should the architect use in order to determine the fire resistance rating of the machine room?

1. Number of stories served by the elevator 2. Fire rating of the associated hoistway 3. Ventilation requirements International Building Code (IBC) 2018, 3005.4 requires the machine room to have the same fire-resistance rating as the elevator hoistway. In order to determine the fire resistance of the hoistway, the architect would need to know the number of stories served by the elevator. The number of stories served by the elevator will determine the fire resistance rating of the hoistway.

How many square inches in a square foot?

144 SQ inch

What is engineered wood span?

15-50'

A studio director is currently estimating the shaft space required for a high-rise office building during the schematic design phase. The typical floor plate is 10,000 square feet. How many square feet of space should be set aside for shaft space?

400 square feet Correct. For high-rise buildings, approximately 4% of the floor area should be set aside for shaft space as a preliminary estimate. This will be refined as the building systems are coordinated with the engineers.

An architect is designing a new 1,200-square-foot house with eight-foot ceilings. The initial design calls for five 3' x 4' windows that allow 60 cfm of fresh air each. However, the local sustainable design ordinance requires new houses to provide three fresh air changes per hour. How many additional 3' x 4' windows are required to comply with the ordinance?

3 CALCULATIONS 1. House volume: 1,200 square feet x 8 foot ceiling height = 9,600 cubic feet 2. Cubic feet per hour required by ordinance: 9,600 cubic feet x 3 fresh air changes per hour = 28,800 cubic feet per hour 3. Cubic feet per minute required: 28,800 cubic feet per hour / 60 minutes per hour = 480 cubic feet per minute 4. Cubic feet per minute provided by five windows: 5 windows x 60 cubic feet per minute = 300 cubic feet per minute 5. Cubic feet per minute deficit: 480 cubic feet per minute required - 300 cubic feet per minute already available = 180 cubic feet per minute still needed 6. Additional windows required: 180 cubic feet per minute / 60 cubic feet per minute (each window) = 3 windows Topic: Project Integration of Program & Systems

An architecture firm is designing a 21-story office building in a metropolitan area. Ten elevators will be provided, based on the amount of occupants calculated to use the space. What is the minimum number of hoistways that can be used?

3 hoistways Correct. Each hoistway can contain four elevators at maximum. IBC 3002.2Number of elevator cars in a hoistway. Where four or more elevator cars serve all or the same portion of a building, the elevators shall be located in not fewer than two separate hoistways. Not more than four elevator cars shall be located in any single hoistway enclosure.

What is the recommended set back to preserve wildlife?

300'-0"

An architect is working with a developer on a new riverside housing project. The project wants to take advantage of the adjacent river and wildlife habitats. What is the minimum recommended width for a buffer between the development and the river to preserve the wildlife habitat?

300'-0" Correct. 300'-0" is the recommended setback to preserve the wildlife habitat. 100'-0" Incorrect. While 100'-0" is the recommended minimum buffer to filter pollutants, 300'-0" is the recommended buffer for a wildlife habitat.

An architect is assessing ADA compliance of an existing elevator in a multi-story residential building. What is the minimum depth and width of the elevator car required by the ADA?

54" x 36" Correct. Per ADA Standard 407.4.1 Car Dimensions, existing elevator car configurations that provide an inside clear depth of 54 inches (1370 mm) minimum and a clear width of 36 inches (915 mm) minimum shall be permitted.

A municipal airport terminal with a gross area of 200,000 sf​​ ​​​​​utilizes a variable air volume (VAV) central HVAC system. How much area should be allocated for all components of the HVAC system?

6,000 square feet Correct. VAV systems are a subset of all-air HVAC systems. All-air systems need about 3-9% of the gross building area for the HVAC system, depending on the building size and program. A good rule of thumb is large buildings (>50,000 sf) set aside 3% of the gross area for HVAC systems, midsize buildings (between 5,000 sf and 50,000 sf) set aside 6%, and small buildings (<5,000 sf) set aside 9%. Since the building is large, taking 3% of the gross area would result in 6,000 sf designated for the HVAC systems. (200,000 square feet x .03 = 6,000 square feet.)

A municipal airport terminal with a gross area of 200,000 sf​​ ​​​​​utilizes a variable air volume (VAV) central HVAC system. How much area should be allocated for all components of the HVAC system?

6,000 square feet Correct. VAV systems are a subset of all-air HVAC systems. All-air systems need about 3-9% of the gross building area for the HVAC system, depending on the building size and program. A good rule of thumb is large buildings (>50,000 sf) set aside 3% of the gross area for HVAC systems, midsize buildings (between 5,000 sf and 50,000 sf) set aside 6%, and small buildings (<5,000 sf) set aside 9%. Since the building is large, taking 3% of the gross area would result in 6,000 sf designated for the HVAC systems. (200,000 square feet x .03 = 6,000 square feet.)

What are called dog bne cuts, wat do they do they do on steel?

In the event of seismic, it deforms so that there is not a catastrophic failure of the welded or bolted joint itself.

A large university building and associated parking lot expansion creates too much stormwater run-off for the constrained site. How should the design team maintain the pre-development rate of flow to avoid sacrificing site or built works?

A detention vault Correct. A stormwater detention vault is used for stormwater storage where there is not enough space for an above-ground facility. The excess stormwater run-off is slowly released to local storm sewers or stream. Underground storage structures are commonly located under parking lots. A detention basin Incorrect. Detention basins provide only flood control measures and are known as dry ponds. The pond is intended to drain the stormwater within a period of time to make the volume available for the next storm event. This approach takes a lot of space and would greatly affect the constrained site.

Which of the following best describes a building material with high thermal resistance?

A porous material A porous medium or a porous material is a material containing pores. The skeletal portion of the material is often called the "matrix" or "frame". The pores are typically filled with a fluid

What is a Catch Basin?

An underground reservoir with a sump built into it; debris settlsttles into the sump instead of flooflowingn the sewer and potentially clogs the system.

An architect has just entered into a contract with a new client. The architect is explaining to their team which design alternative studies are included in the scope of basic services and which should be performed as an additional service. Which of the following design alternative studies are covered under the scope of the architect's basic services, per the AIA Standard Form of Agreement between Owner and Architect?

Alternate building systems Correct. Per AIA B101-2017 Article 3.2.5.2, the architect shall consider alternative building systems as part of the basic services. Inclusion of design alternates in specifications Correct. Per AIA B101-2017 Article 4.2.1.6, the architect shall include design alternatives for bidding purposes in the contract documents, which include the specifications. Alternative materials Correct. Per AIA B101-2017 Article 3.2.5.2, the architect shall consider alternative materials as part of the basic services. Sustainable design alternatives Correct. Per AIA B101-2017 Article 3.2.5.1, the architect shall consider sustainable design alternatives as part of the basic services. Multiple preliminary designs Incorrect. Per AIA B101-2017, this is listed as an additional service.

A museum building is listed on the National Register of Historic Places and is undergoing alterations to provide a better experience for guests. An architect is reviewing accessibility requirements as part of the alteration to provide wheelchair access to the second floor of the museum. Which of the following statements is true regarding the alterations?

Adding an elevator inside the building structure will significantly impact the historic interior and threaten its historic significance, therefore, alternative access can be considered Correct. Adding an elevator inside the building structure will significantly impact the historic interior and threaten its historic significance; therefore, alternative access can be considered. Alterations to buildings listed in the National Register of Historic Places shall comply to the maximum extent of the law with no exceptions Incorrect. In the 2010 ADA Standards for Accessible Design, section 36.405 provides exceptions to maximum compliance: "If it is determined that it is not feasible to provide physical access to an historic property that is a place of public accommodation in a manner that will not threaten or destroy the historic significance of the building or the facility, alternative methods of access shall be provided...."

The trust that owns a historic bank property is looking to add a green roof to the existing flat-roofed building. A preliminary feasibility study determined that the addition of the green roof would not be visible above the existing parapet, and would require only minor structural work in adding metal plate reinforcement to the existing exposed wood beam structure. The trust has hired an architect to evaluate the approach in line with the standards and best practices of historical rehabilitation. Per the Secretary of the Interior's Standards for Rehabilitation, what would be the best recommendation for the green roof proposal?

Advise the client to investigate alternatives that will not require structural changes Correct. Since the wood beams are exposed, they are considered a historic element, and adding reinforcement would alter the historic nature of the property. Due to the existing structural constraint, the architect should advise the owner to explore alternatives to avoid damaging the historic nature of the structure. Advise the client to proceed and add additional wood beams mimicking the original wood beams, in lieu of metal plates Incorrect. This is not a recommended approach as it would alter the historic nature of the property. Historic mimicry is primarily recommended for replacement of severely deteriorated elements, and is not a recommended method for additions or alterations.

An architect has just entered into a contract with a new client. The architect is explaining to their team which design alternative studies are included in the scope of basic services and which should be performed as an additional service. Which of the following design alternative studies are covered under the scope of the architect's basic services, per the AIA Standard Form of Agreement between Owner and Architect?

Alternate building systems Correct. Per AIA B101-2017 Article 3.2.5.2, the architect shall consider alternative building systems as part of the basic services. Alternative materials Correct. Per AIA B101-2017 Article 3.2.5.2, the architect shall consider alternative materials as part of the basic services. Sustainable design alternatives Correct. Per AIA B101-2017 Article 3.2.5.1, the architect shall consider sustainable design alternatives as part of the basic services. Inclusion of design alternates in specifications Correct. Per AIA B101-2017 Article 4.2.1.6, the architect shall include design alternatives for bidding purposes in the contract documents, which include the specifications. Landscaping design alternatives Incorrect. Per AIA B101-2017, landscaping design in general is listed as an additional service; therefore, alternative landscaping designs would also be an additional service. Multiple preliminary designs Incorrect. Per AIA B101-2017, this is listed as an additional service.

What are the five elements of Passive Heating?

Aperture (Collector) Absorber Thermal mass Distribution Control. A passive solar heating system is made up of the following key components, all of which must work together for the design to be successful: How Does It Work? Passive solar buildings are designed to let the heat into the building during the winter months, and block out the sun during hot summer days. This can be achieved by passive solar design elements such as shading, implementing large south-facing windows, and building materials that absorb and slowly release the sun's heat.

What do architects use a Faraday cage for?

Architects use Faraday cages as lightening protection. By creating a wire mesh on the roof, they can redirect lightening strikes around the building to the ground.

Modern material-testing standards have revealed that several common building materials are not safe for the installer or the occupant of a building. Congressional acts have prohibited the use of dangerous building materials. Which one of the following building materials is prohibited according to the Clean Air Act of 1970 and the Toxic Substances Control Act of 1976?

Asbestos Correct. Asbestos was a common building material, but these acts now regulate it. It requires special attention when discovered. Lead paint Incorrect. The Consumer Product Safety Commission banned lead paint in 1977.

An architect is developing the code review documents for a new medical facility. While performing the code review, the architect notices that certain types of medical facilities are classified as an Institutional occupancy and will require additional considerations. Which of the following medical facilities would be considered an Institutional occupancy?

Assisted living facility Correct. An assisted living facility is an I-1 Institutional occupancy. Rehabilitation facility Correct. A rehabilitation facility is classified as an I-1 Institutional occupancy. Hospitals Correct. A hospital is classified as an I-2 Institutional occupancy. Outpatient clinics Incorrect. An outpatient clinic is classified as a Business (B) occupancy. Ambulatory care facility Incorrect. An ambulatory care facility is classified as a Business (B) occupancy. Veterinary clinic Incorrect. A veterinary clinic is classified as a Business (B) occupancy.

An architect is reviewing the geotechnical report for a new construction project and notices that the groundwater table is noted at 5 feet below grade. The client included a full cellar in the program that they provided to the architect. The architect has been coordinating with the structural engineer to design a building with a cellar slab at 10 feet below grade, enclosed with concrete masonry unit walls. Which of the following should the structural engineer and architect include in their foundation details based on the groundwater table height?

Bentonite clay at wall-to-floor joint Correct. Bentonite clay is a common waterstop used at wall-to-floor joints in foundations subject to potential groundwater intrusion due to the location of the water table.

An architect is reviewing the geotechnical report for a new construction project and notices that the groundwater table is noted at 5 feet below grade. The client included a full cellar in the program that they provided to the architect.The architect has been coordinating with the structural engineer to design a building with a cellar slab at 10 feet below grade, enclosed with concrete masonry unit walls. Which of the following should the structural engineer and architect include in their foundation details based on the groundwater table height?

Bentonite clay at wall-to-floor joint Correct. Bentonite clay is a common waterstop used at wall-to-floor joints in foundations subject to potential groundwater intrusion due to the location of the water table.

A client is interviewing architectural firms regarding their experience and knowledge of evolving sustainable design strategies and technologies. Which strategies or technologies should the firm present to the client during the interview?

Building Integrated Photovoltaics (BIPV) Correct. BIPV is an evolving solar energy technology that architects should be familiar with. The technology embeds photo-voltaic solar cells in the glazing itself, thus maximizing the surface area of buildings for electricity generation. Carbon-neutral design Correct. Climate change and global warming are growing concerns in the design community. Carbon-neutral design strategies include early design phase studies of energy use intensity (EUI) and embodied carbon in building materials which measures the carbon dioxide emissions from materials from cradle to gate. This refers to when the building material is extracted, manufactured, distributed, installed, operated, and then recycled, reused, or salvaged. Regenerative design Correct. The goal of regenerative design is to produce a net positive environmental impact with respect to energy, water, and materials. This design strategy is new and evolving. Systems include biologically engineered wastewater treatment using living plant species to treat water, generate biomass in aquacultural ponds, promoting green roofs for stormwater management, carbon sequestration, and fauna habitat. Passive solar design Incorrect. Passive solar design has been around for centuries. The Greeks are considered to be among the first civilizations to design buildings for passive heating potential. Many of the same passive design strategies can be adopted in current design strategies, including building orientation, thermal massing, water harvesting, solar gain, and natural ventilation. Ground-sour ce heating and coolingIncorrect. Ground-source heating and cooling systems have been around for decades. Ground source heat pumps are different from geothermal energy, as geothermal energy uses the heat generated from the core of the earth. Ground source heat pumps, however, use the sun's energy, which is stored in the ground as heat

The architect and structural engineer for a 73-story apartment building are evaluating the options for the structural framing system for the building. Because of the building's height, they need a system that will resist the lateral forces imposed on the structure. Which of the following framing systems would be appropriate for this high-rise structure?

Bundled tube Correct. A bundled tube structure ties narrow vertical tubes together to form a modular structure that is appropriate for high-rise buildings. Diagrid Correct. A diagrid is a diagonal grid frame where the diagonal members are connected at specially jointed nodes. This exterior framework has the potential for reducing the number of internal supports. It provides both shear and bending rigidity and is highly effective in resisting the lateral forces of a high-rise building. Trussed tube Correct. A trussed tube has trussed wall frames with widely spaced columns that are tied together with either diagonal or cross bracing, making it appropriate for high-rise construction. Latticed truss Correct. A latticed truss has perimeter frames of closely spaced diagonals with no vertical columns, and is appropriate for high-rise buildings. Space frame Incorrect. A space frame is made up of short rigid linear elements that are triangulated in three dimensions. It is not an efficient structural framing system for a high-rise building.

An architect must evaluate the ADA conformance of passenger elevators located in an existing office building. Which of the following statements accurately represents the ADA guidelines on existing passenger elevators?

Car control buttons are permitted to be recessed Correct. Per ADA Standard 407.4.6.2, in existing elevators, buttons shall be permitted to be recessed. Car gates are prohibited Incorrect. Per ADA Standard 407.3.2, existing elevators with manually operated swing doors shall be permitted.

An architect is designing a multi-story warehouse. The warehouse is to be constructed on a site where zoning regulations limit the height. Which structural system is most appropriate?

Cast-in-place, reinforced concrete columns, capital, drop, and flat slab floors.

Where are castellated beams used?

Castellated beams used in construction are highly effective structural elements found in long span floor and roof systems. Their high flexibility, reduced weight and great aesthetics make them optimal picks for low-cost solutions. While every project is different, most benefit from such a combination.

An architect is performing a code analysis for a new 15,700-square-foot building which will predominately be classified as type A occupancy. The building will have four 390-square-foot storage rooms that will be type S-2 occupancy. Which of the following statements regarding the store rooms is true?

Cannot exceed 10% of the total floor area allowed by the height/area table and does not need to be separated from the main area by a fire separation Correct. This situation is considered accessory use because the S-2 occupancy is less than 10% of the A occupancy. No separation is required between accessory occupancies and the main occupancy.

What is cathodic protection?

Cathodic protection is a technique used to control the corrosion of a metal surface by making it the cathode of an electrochemical cell. A simple method of protection connects the metal to be protected to a more easily corroded "sacrificial metal" to act as the anode.

An architect is designing a new 1,200-square-foot house with eight-foot ceilings. The initial design calls for five 3' x 4' windows that allow 60 cfm of fresh air each. However, the local sustainable design ordinance requires new houses to provide three fresh air changes per hour. How many additional 3' x 4' windows are required to comply with the ordinance?

CORRECT RESPONSE 3 CALCULATIONS 1. House volume: 1,200 square feet x 8 foot ceiling height = 9,600 cubic feet 2. Cubic feet per hour required by ordinance: 9,600 cubic feet x 3 fresh air changes per hour = 28,800 cubic feet per hour 3. Cubic feet per minute required: 28,800 cubic feet per hour / 60 minutes per hour = 480 cubic feet per minute 4. Cubic feet per minute provided by five windows: 5 windows x 60 cubic feet per minute = 300 cubic feet per minute 5. Cubic feet per minute deficit: 480 cubic feet per minute required - 300 cubic feet per minute already available = 180 cubic feet per minute still needed 6. Additional windows required: 180 cubic feet per minute / 60 cubic feet per minute (each

A client asks an architect to perform a feasibility study for a rectangular 50' x 100' urban lot. The 25-foot side fronts a public right-of-way. The zoning ordinance stipulates the following: Front and side setbacks: 5' Rear setback: 25' Maximum impervious coverage: 2,400 sf Floor Area Ratio (FAR): 2.0 (based on total land area, including setbacks) Max building stories on site: 4 What is the maximum gross building area?

CORRECT RESPONSE 9,600 CALCULATIONS 1. Front and rear setbacks: 100 feet - 5 feet - 25 feet = 70 feet 2. Side setbacks: 50 feet - 5 feet - 5 feet = 40 feet 3. Buildable site area within setbacks: 40 feet x 70 feet = 2,800 square feet However, maximum impervious coverage is 2,400 square feet so this becomes the footprint's limiting factor. 4. Calculate maximum FAR: 50 feet x 100 feet = 5,000 square feet (total land area) x 2 (FAR) = 10,000 square feet 5. Maximum gross floor area: 2,400 square feet (max. impervious coverage) x 4 stories = 9,600 square feet (which is under the maximum FAR)

A low-rise building is designed to allow for a future addition that will result in a midrise building with a higher water demand. When the addition is complete, the majority of the building's rooftop will be dedicated to the mechanical system and individual floors will not be able to support a water tank. Which water supply system should be used?

CORRECT RESPONSE Pumped upfeed distribution Pumped upfeed distribution uses multiple pumps to supply water directly to fixtures without using a water storage tank.

Drag the building systems into the boxes on the psychrometric chart to indicate the system that is most appropriate for each climate condition. Not all building systems will be used.

CORRECT RESPONSES Ceiling Fans Ceiling fans are a form of evaporative cooling, which is recommended in System A. Operable Windows Operable Windows are a form of natural ventilation, which is recommended in System B. Roof Ponds Roof ponds are a form of high-mass cooling, which is recommended in System C. Topic: Building Systems, Materials, & Assemblies

A municipality hires an architect to design a public park in an environmentally sensitive area. The architect is designing a pedestrian pathway and wants to specify a material that maximizes infiltration. Which material should the architect specify?

Crushed stone My original answer was concrete pavers thinking ADA access should be required. But since there is no mention of this in the question. This thought should be omitted.

An architect is designing the fire suppression system in a new data center. In addition to an overhead system, the building department requires portable fire extinguishers to be spaced evenly throughout the server room in the event of a fire. What type of portable fire extinguisher is required for this application?

Class C Correct. Class C fire extinguishers should be used for fires involving energized electrical equipment. Class A Incorrect. Class A fire extinguishers should be used for fires involving ordinary combustibles such as wood, cloth, paper, rubber, and many plastics. Class B Incorrect. Class B fire extinguishers should be used for fires involving flammable or combustible liquids, gases, or greases. Class D Incorrect. Class D fire extinguishers are used with fires that involve combustible metals or metal alloys.

During design discussions for a new bank in Minnesota, several solutions have been offered for how to slow traffic in the approach to the drive-through teller window. The client is interested in increasing the aesthetic value of the exterior of the bank in order to make it as inviting as possible to the public. Which site improvement would most appropriately slow down the traffic?

Colored brick pavers in the drive-up lane area Correct. Brick pavers offer both a visual and auditory cue to drivers and they also provide aesthetic value. The pavers have an added benefit of allowing stormwater to pass through their joints, reducing runoff. Speed bumps at each of the drive-up lane entrances Incorrect. Speed bumps are effective for traffic slowing, but should not be chosen here because they encumber snow removal.

An architect is designing a new addition to an existing college student center. The addition is to include a new lecture hall, dining hall, and pre-function space. The architect has decided to design the building with an exposed rigid steel frame to keep with the industrial feel of the existing college student center. At one time, the structure was a train station and still features the original steel framing. What challenges might this system present to the architect?

Columns may be required where undesirable Correct. A rigid steel frame may require more columns than other, longer spanning systems in such areas as the lecture hall. Cost of construction may be high Correct. Steel members can be very costly compared to other systems utilizing concrete, wood stud framing, or cold formed metal framing. Providing required fire ratings may be costly Correct. The building will likely have required fire ratings for the structure based on the use and typical size of the programs included. Fireproofing steel while leaving it exposed is often very costly. Required level of finish may be difficult to achieve Incorrect. The prompt states that the exposed steel is chosen to preserve the industrial look of the existing facility. Therefore, the level of finish is achieved through the material itself, with minimal additional work required

An architect is working with their partner to determine which types of services need to be performed on the various project types handled by their office. Which of the following projects is most likely to require a master plan?

Community college expansion Correct. Master plans are typically provided when the client is a large organization and when the project includes a variety of buildings, uses, and spaces. A college expansion likely meets these criteria. Urban infill mixed-use building Incorrect. Master plans are typically provided for large sites with a variety of buildings and uses. A mixed-use building on an urban infill lot does not meet these criteria.

The owner of a library located in a cold rainy climate, has asked the architect for suggestions for cost-effective paving material for a new outdoor reading plaza. Which one of the following paver materials should the architect suggest to the owner?

Concrete Correct. Concrete is a cost-effective solution and can be prepared to have a more slip-resistant surface for adverse weather conditions. Flagstone Incorrect. Since the site is in an area where it may rain often, flagstone is not a good option because it can become slippery when wet. Flagstone is often moderately priced, and more cost-effective options can be chosen.

An architect is designing a 15-story office building. Due to adverse soil conditions, the structural engineer advises that the building must be supported on piles that extend 200 feet into the soil. Which type of piles should the architect recommend?

Concrete-filled drilled caissons Correct. Concrete-filled drilled caissons can extend up to 250 feet into the earth. They also have the highest bearing capacity of all pile types.

Two bodies in direct contact with each other will transfer heat primarily by the mechanism of ________.

Conduction

What is conduction?

Conduction is the process by which heat is transferred from the hotter end to the colder end of an object. Heat spontaneously flows from a hotter body to a colder body. For example, heat is conducted from the hotplate of an electric stove to the bottom of a saucepan in contact with it

An architect is specifying Douglas fir 2x12 rafters at 24" on center for a new single-family addition. In order to obtain the Douglas fir rafters, what kind of tree would the wood need to be sourced from?

Coniferous Correct. Douglas fir is a softwood that is sourced from coniferous trees.

A developer is looking to construct a new mixed-use property in a local business district. Several older properties will be demolished to make room for the new property. Early in design, an environmental impact study was conducted to understand the impact of the development, and it was determined that several of the existing buildings have historical significance. Following the impact study, which of the following would be appropriate actions?

Consider relocating the project site Correct. Depending on the significance of the existing structures, it may be more beneficial to choose an alternate site for the project and preserve the existing properties. Incorporate features of the existing buildings Correct. By incorporating features of the existing buildings to be demolished into the new project, the potential negative impacts of the project can be minimized. Design the new building in a way that enhances the buildings to remain Correct. The new building should compliment the existing buildings or enhance the character of the properties that remain. Document the existing buildings Correct. By completing measured drawings and taking archival-quality photos of the buildings to be demolished, these historic properties can be documented and archived with a narrative about their historical significance. Sell historically significant artifacts to historical society Incorrect. Selling artifacts to the historical society instead of donating them could negatively affect the public perception of the project.

A developer is looking to construct a new mixed-use property in a local business district. Several older properties will be demolished to make room for the new property. Early in design, an environmental impact study was conducted to understand the impact of the development, and it was determined that several of the existing buildings have historical significance. Following the impact study, which of the following would be appropriate actions? Check the four that apply.

Consider relocating the project site Correct. Depending on the significance of the existing structures, it may be more beneficial to choose an alternate site for the project and preserve the existing properties. Incorporate features of the existing buildings Correct. By incorporating features of the existing buildings to be demolished into the new project, the potential negative impacts of the project can be minimized. Design the new building in a way that enhances the buildings to remain Correct. The new building should compliment the existing buildings or enhance the character of the properties that remain. Document the existing buildings Correct. By completing measured drawings and taking archival-quality photos of the buildings to be demolished, these historic properties can be documented and archived with a narrative about their historical significance. Sell historically significant artifacts to the historical society Incorrect. Selling artifacts to the historical society instead of donating them could negatively affect the public perception of the project.

Which type of HVAC system is indicated in the schematic diagram?

Constant air-volume The diagram depicts a single duct constant air volume system. This system delivers conditioned air at a constant temperature through a low-velocity duct system. Topic: Building Systems, Materials, & Assemblies

An architect is responsible for the acoustical design of a mental health clinic. The space holds 32 private offices for counselors to conduct one-on-one sessions with their clients. Each counselor's office needs to be acoustically separated for privacy reasons. Which of the following features should the architect incorporate into the design?

Continuous gaskets around doors and caulking around door frames Correct. Door openings provide potential locations for sound transfer. Staggered electrical outlets within partitions so they are not back-to-back Correct. Back-to-back outlets provide potential openings for sound transfer and should be avoided. Acoustical sealant at the junctions where walls meet ceilings and floors Correct. Providing airtight construction will reduce sound transfer. High-mass materials such as solid concrete for the wall construction Correct. High-mass construction is better at reducing sound transmission. This is because sounds are vibrations and high-mass materials will not vibrate as readily. Hard gypsum board ceiling and sealed concrete floors Incorrect. Limp materials provide better sound reduction than stiff materials.

An architect is in the programming phase of a tenant improvement project for a small university office suite in an existing building. The client has asked for five 120-square-foot offices for staff members, a 200-square-foot conference room, and a 100-square-foot storage room. The client has a limited budget and would like to minimize the area to be remodeled. Other programming requirements follow: Use a building efficiency ratio (net-to-gross) of 0.75 if a double-loaded corridor design is implemented. Use a building efficiency ratio (net-to-gross) of 0.70 if a single-loaded corridor design is implemented. At least two offices and conference rooms should be located along exterior walls for views. The dimension of the longest length of existing space to remodel is 50 feet. How much area is needed for the renovation? _______ square feet

Correct answer: 1200 CORRECT RESPONSE 1,200 square feet A double-loaded corridor design should be implemented to minimize the area that needs to be remodeled. CALCULATIONS 1. Total office area: 120 sf x 5 offices = 600 sf 2. Total net square footage: 600 sf (offices) + 200 sf (conference room) + 100 sf (storage room) = 900 nsf 3. Gross square footage needed: 900 net sf ÷ 0.75 efficiency ratio = 1,200 sf Section: Project Integration of Program & Systems

An architect is designing a new nature center. A large, paved plaza in front of a multi-vehicle drop-off area is being planned at the main entrance of the project. What key considerations should the architect make when designing the plaza?

Designing the plaza in a way that will allow runoff to be collected in a constructed stormwater wetland Correct. Constructed stormwater wetlands are designed to resemble natural environments and would be appropriate for the nature center. Utilizing a rigid paver system Correct. Rigid paver systems effectively shed water when installed correctly. Utilizing a rigid paving that collects water and then flows to a swale or stormwater wetland is a common solution to stormwater management. This system allows for a high level of control in terms of where water goes when it rains. Providing stormwater collection tanks under the plaza Incorrect. This strategy is often used for large parking lots when no other method of stormwater control is possible. Integrating a detention basin into the plaza design Incorrect. A detention basin holds large amounts of stormwater runoff and is unlikely to be allotted such a large area in an entry plaza.

The owner has requested additional large storefront windows along the front facade of their new school after schematic design review. The current facade is brick with metal stud backup. Which of the following CSI sections of the cost estimate should be updated to reflect the changes?

Division 04 Correct. Division 04 - masonry of the cost estimate would need to be changed to reflect the reduction of brick needed due to the additional windows. Division 08 Correct. Division 08 - Openings would need to reflect change in cost for the additional number of windows.

An architect is designing a two-story surgery center with an attached drive-up canopy to protect visitors from rain and snow. They have determined from the code review that an automatic sprinkler system will be required for the entire building, including the drive-up canopy. What type of sprinkler system should be specified for the canopy?

Dry-pipe system Correct. Since the building is located in an area subject to freezing (as the scenario states that there can be snow), a dry-pipe system should be specified for the canopy. The pipe in this system contains pressurized air instead of water. In the event of a fire, the air is released when the spinkler heads open to allow water to flow through the pipe and out the nozzle. Wet-pipe system Incorrect. A wet-pipe system contains water in the pipes and should not be used in areas that are subject to freezing.

An architect is designing a replacement roofing system for an exiting building in a hospital complex. The hospital is worried about the effects of the installation of the roof on patienets with breathing difficulties. The hospital has asked the architect to choose a roofing system that minimizes health risks. Which roofing system should the architect specify?

EPDM EPDM membranes have a minimal amount of seams which are joined together with adhesive. This roofing system is the most appropriate option because it is the only system that will not pose a risk to the health of hospital platiemnts during isntallation. patients

What is an EPDM roofing system?

EPDM is an extremely durable synthetic rubber roofing membrane (ethylene propylene diene terpolymer) widely used in low-slope buildings in the United States and worldwide. Its two primary ingredients, ethylene, and propylene, are derived from oil and natural gas.

An architect is designing a new subdivision on a large plot of land adjacent to a commuter train route. What is the most effective option for mitigating off-site noise intrusion from a nearby train route?

Earthworks to block sound Correct. The best approach is to adjust the grading to deaden the noise. Construct sound barriers Incorrect. Sound barriers do not have enough mass to effectively deaden the noise from a train route.

The design of a new skyscraper has multiple floors below grade. The second level below grade will have restrooms and a staff room for the maintenance team. This level will be below the main sanitary sewer to which the plumbing fixtures must be connected. Which of the following plumbing elements must be included in the design

Ejector pump Correct. An ejector pump looks similar to a sump pump, but its job is to collect sewage from the basement plumbing that is lower than the municipal sewer line and pump it up and out. The ejector pump is in a pit in the ground, like a sump pump. It has a lid and is vented. Sump pump Incorrect. A sump pump is not for use with sewage. Sump pumps are for pumping groundwater from the drain tile piping or for pumping water out of the basement.

A university has mandated that all new campus projects must achieve LEED certification. Which method of energy calculation should be used by the project team?

Energy cost budget Correct. The energy cost budget method is used to verify compliance with LEED. It determines the energy budget for a specific building. The budget is calculated with a computer simulation of hourly energy use over the course of a year. Although it is a complex method, it is the only way to deal with individual projects and the unique aspects of a design. Building area Incorrect. The building area method provides a maximum allowable power in watts per square foot of building area, based on the building type. This may be used to meet several code requirements. However, it is not the prescribed method for LEED certification, which requires energy modeling.

A university has mandated that all new campus projects must achieve LEED certification. Which method of energy calculation should be used by the project team?

Energy cost budget Correct. The energy cost budget method is used to verify compliance with LEED. It determines the energy budget for a specific building. The budget is calculated with a computer simulation of hourly energy use over the course of a year. Although it is a complex method, it is the only way to deal with individual projects and the unique aspects of a design. Space-by-space Incorrect. The space-by-space method assigns allowable wattage per square foot to spaces based on their use type. When multiplied by the gross area of each programmatic space, a lighting power density can be determined and compared to the code requirements. However, it is not the prescribed method for LEED certification, which requires energy modeling.

The parts of a packaged unit HVAC system are listed below. In which stage of the refrigeration cycle is the refrigerant a gas under low pressure?

Evaporator Correct. The question is describing an evaporative process in which the refrigerant turns from a liquid to a vapor; thus the evaporator is the correct answer. Condenser Incorrect. The condenser is where the refrigerant turns back into a liquid.

What is a staggered truss system?

Everyone other floor is a floor, each truss is going to hold the floor below and above it. We could have column-free floor plates.

An architect is working on the design of an exterior wall system for a proposed auto repair garage. The site is a narrow lot, and the architect is exploring exterior wall systems that can provide the required thermal resistance while taking up the least amount of space. The architect has determined that they need to provide R-18 insulation within a 3-inch-thick space in order to achieve the desired result. Which of the following insulation types meet the design criteria? Check the two that apply.

Expanded polyurethane Correct. Expanded polyurethane provides an R-value of approximately 6.2 per inch. This would yield a total R-value for the insulation of R-18.6 (R-6.2/in x 3 inches), which is above the R-18 requirement. Polyisocyanurate Correct. Polyisocyanurate provides an R-value of approximately 7.2 per inch. This would yield a total R-value for the insulation of R-21.6 (R-7.2/in x 3 inches), which is above the R-18 requirement. Molded polystyrene Incorrect. Molded polystyrene provides an R-value of approximately 3.6 per inch. This application requires an R-value per inch of at least 6.0 (R-18 / 3 inches = 6.0 minimum). Cellulose Incorrect. Cellulose provides an R-value of approximately 3.7 per inch. This application requires an R-value per inch of at least 6.0 (R-18 / 3 inches = 6.0 minimum). Rock wool Incorrect. Rock wool provides an R-value of approximately 3.3 per inch. This application requires an R-value per inch of at least 6.0 (R-18 / 3 inches = 6.0 minimum). Extruded polystyrene Incorrect. Extruded polystyrene provides an R-value of approximately 5.0 per inch. This application requires an R-value per inch of at least 6.0 (R-18 / 3 inches = 6.0 minimum).

An architect is working with a developer to build single-family homes that are in the flight path of a nearby airport. Which of the following items would offer the best opportunities to mitigate noise in the houses coming from airport operations? Check the four that apply.

Exterior wallboard with resilient fasteners Correct. Attaching wallboard to exterior wall studs with resilient fasteners or clips reduces sound transmission through the walls. Central air conditioning and heating Correct. A key factor in airport noise mitigation is keeping windows and doors closed. Central air conditioning and heating will provide year-round fresh air ventilation while allowing the windows to be closed. Triple-pane windows Correct. Triple-pane windows add extra air spaces between panes of glass that can help mitigate airport noise. Baffled vents Correct. Airport noise can enter the home through exterior vents in the house, such as exhaust fans and crawl space vents. Providing sound baffles in these vents can help mitigate Interior wall insulation Incorrect. While installing interior wall insulation can reduce noise levels from room to room within the house, it will not mitigate noise coming from the outside of the house. Reduced air space in exterior walls Incorrect. To mitigate sound through exterior walls, it is better to increase the air space in the wall.

Controlling summer solar heat gain through windows is critical to reducing cooling demand and improving occupant comfort. When properly designed, which of the following are effective solutions to reduce summer solar heat gain through south-facing windows? Check the three that apply.

Exterior window awnings Correct. Exterior window awnings can block 100% of unwanted solar heat gain through windows when accurately designed for the high angle of the summer sun. Exterior motorized window shades Correct. Exterior motorized window shades can block 100% of unwanted solar heat gain through windows. Roof overhangs Correct. Exterior roof overhangs can block 100% of unwanted solar heat gain through windows when accurately designed for the high angle of the summer sun. Increase the glass R-value Incorrect. Increasing the R-value of the insulated glass unit does little to reduce direct solar heat gain. Decreasing the solar heat gain coefficient or using exterior shading elements is significantly more effective.

Fire wall Vs Fir Barrier

Fire Wall Most stringent-you essentially build two different buildings, structurally independent of one another. Separated two construction types or two different areas. Extends from foundation through roof Fire Barrier More commom-extends from floor to underside of structural ceiling and does not need to be structurally independent. Required fro mechanical shaft walls, egress stair walls, separated uses, and incidental uses. Fire partition Least stringent-can be mad out of wood and can include a dropped ceiling. (Doesn't have to extend to the deck above) Required for corridors.

An architect is developing plans for a new electric vehicle manufacturing facility. Early in the design process, the architect requests a meeting with the building plan reviewer to discuss the project and identify any concerns the reviewer may have before moving forward with the project design. In addition to the building plan reviewer, who else should the architect recommend attend the meeting?

Fire chief Correct. The fire chief would be a beneficial addition to the meeting as the building inspector and fire plan reviewer sometimes have different interpretations of the building and fire codes. Also, the fire chief may have additional concerns based on the current capacity of the fire department.

An architect for a wood-framed custom residence has designed a beam to support a portion of the second floor above the garage. The beam will consist of two steel plates sandwiched between three pieces of solid-sawn 2x12 lumber that are bolted together. What type of beam has the architect designed?

Flitch beam Correct. A flitch beam is a series of one or more steel plates bolted in between wood structural member Built-up beam Incorrect. A built-up beam consists of two or more wood members that are nailed together. Spaced beam Incorrect. A spaced beam consists of two wood members that are spaced apart with blocking at specific intervals and nailed securely together. Box beam Incorrect. A boxed beam consists of two or more plywood plates glued to either side of wood flanges at the top and bottom.

An engineer is tasked with designing a mechanical system in a building with minimal space. To save space a single system will be used. Which HVAC systems are capable of both heating and cooling with the same units? Check the four that apply.

Forced air Correct. Forced air systems use a furnace, for heating, and direct expansion, for cooling. Through-wall units Correct. Through-wall units are similar to packaged terminal units. They provide cooling, through compressive refrigeration, and heating, through either electric resistance or by reversing the refrigeration cycle. Packaged terminal units Correct. Packaged terminal units (PTACs) are similar to through-wall units. They provide cooling, through compressive refrigeration, and heating, through either an electric resistance or by reversing the refrigeration cycle. Heat pumps Correct. Heat pumps can be reversible. They can pump heat out (cooling) or pump it in (heating). Packaged evaporative air Incorrect. Evaporative units can only cool. Radiant panels Incorrect. Radiant panels are for heating only.

Which HVAC sstems are capable of both heating and cooling with the same units?

Forced Air Packaged Terminal Units (PTACs) Trough-wall units Heat Pumps Heat pumps can be reversible. They can pump heat out (cooling) or pump it in (heating)

A national home builder is analyzing the residential heating and cooling systems that they incorporate into their houses in different parts of the country. They want to minimize the system operating costs and energy consumption in their cold climate homes. Which of the following would be the best system choice to meet these goals?

Forced air Correct. Forced air heating and air-conditioning units are fuel-efficient, versatile, and widely used for residences. Electric baseboard convector Incorrect. Electrical baseboard convectors can be an efficient means of heating where electricity costs are low. However, cooling must be provided by a separate system at additional cost.

What is a GFCIT plug?

GFCI stands for Ground Fault Circuit Interrupter. These are also referred to as GFIs, or Ground Fault Interrupters. A GFCI precisely monitors the balance of electrical current moving through a circuit. If the power goes where it shouldn't, like in a short, the GFCI immediately cuts off the electricity The NEC requires GFCIs on all exterior and bathroom receptacles (another term for outlets). GFCIs are also required on all receptacles serving kitchen countertops. In laundry rooms and utility rooms, GFCIs should be installed on outlets within six feet of sinks, washing machines, and water heaters.May 18, 2022

An architect is working on selecting an elevator for a new luxury high-rise apartment building. The elevator needs to be a quiet, high-speed, and energy-efficient system able to handle the day-to-day needs of the residents. Space is also at a premium within the building, so a machine-room-less system is preferred. Which of the following elevator types would be most appropriate given the requirements?

Gearless traction elevator Correct. While the gearless traction elevator provides the same performance as a geared traction elevator, a gearless traction elevator is quieter, more efficient, and does not have a gearbox, which eliminates the potential need for a machine room. Geared traction elevator Incorrect. While a geared traction elevator would be appropriate given its speed and height capabilities, the required gearbox is noisier and would require more space (i.e. a machine room) than the gearless option.

What is galvanic action?

Galvanic action occurs when two electrochemically dissimilar metals are in contact and a conductive path occurs for electrons and ions to move from one metal to the other. One metal corrodes as its ions are deposited onto the other metal

An architect is working on selecting an elevator for a new luxury high-rise apartment building. The elevator needs to be a quiet, high-speed, and energy-efficient system able to handle the day-to-day needs of the residents. Space is also at a premium within the building, so a machine-room-less system is preferred. Which of the following elevator types would be most appropriate given the requirements?

Gearless traction elevator Correct. While the gearless traction elevator provides the same performance as a geared traction elevator, a gearless traction elevator is quieter, more efficient, and does not have a gearbox, which eliminates the potential need for a machine room. Incorrect. While a geared traction elevator would be appropriate given its speed and height capabilities, the required gearbox is noisier and would require more space (i.e. a machine room) than the gearless option.

What is a girder, beam, joist relationship?

Girder- shorter span, beam -longer direction, joist-in between beam- decking-always laid in the opposite directio

Peak demand for cooling often coincides with peak demand for electricity. This results in a spike in electricity use when electricity is most costly. Which type of cooling flattens this curve?

Ice storage Correct. An ice storage cooling system uses energy overnight, when it is less expensive, to freeze ice. The ice is then used during the day to cool the building.

A seven-story structure is being planned. The structural system must minimize on-site construction time, and the floor thickness should be as thin as possible. Which structural system should the architect select?

Hollow core slab Correct. A hollow core slab minimizes site construction time and allows for thin floor thickness. Post-tensioned one-way slab Incorrect. Post-tensioning does allow for a thinner floor slab thickness, but does not decrease the on-site construction time.

What is a Vierendeel truss?

noun. Vie·​ren·​deel truss. ˈvirənˌdāl- variants or Vierendeel girder. : an open-web truss with vertical members but without diagonals and with rigid joints.

An electrical engineer wants to look up the minimum required number of foot candles for interior artificial lighting. Where should they look?

IBC Chapter 12 addresses the Interior Environment. Note: Chapter 27 addresses electrical, the requirements outlined in this chapter are for general electrical supply and emergency and standby power systems.

Where do you locate your building on a hill? At the bottom? Partway up? At the crest?

If you are building in the mountains, know that the valley will pool sinking cold air on still nights (good for hot-arid) and the top will have the most wind (good for hot humid). So locate your buildings on the following elevations based on climate:

What is ballon framing?

In balloon framing, the studs (vertical members) extend the full height of the building (usually two stories) from foundation plate to rafter plate, as contrasted with platform framing, in which each floor is framed separately.

What is platform framing?

In platform framing, each floor is framed separately, as contrasted with balloon framing, in which the studs (vertical members) extend the full height of the building. Freed from the heavy timbers of the post-and-beam system, platform framing offers ease of construction.

What is the difference between A Shear Wall Connection and a Moment Connection?

In steel, you can recognize a shear connection because (generally) the beam web is bolted or welded to the column, but beam's flanges are not. Shear walk connections resist gravity, but don't do well in the presence of lateral forces like wind and seismic. Therefore they need additional lateral resistance from cross bracing or a shear wall. By contrast steel moment connections (generally) bolt/welded both the flanges and web to the column and resist both vertical f=gravity and lateral wind/seismic.

The owner would like to create typical live/work apartments consisting of one bedroom and one living room on the upper floors of the existing building. The apartments will each have 300 sf of open living space and 300 sf of bedroom space, excluding the bathroom and kitchen. The architect is verifying operable glazing requirements against the existing window characteristics. Based on the information provided, what is the area of operable glazing required per room to provide fresh air?

Incorrect - the answer is 15 The provided reference states that any habitable room must have glazing with an area no less than 8% of the floor area of that space, and must have operable window area totaling a minimum of 5% of the floor area of the space. The planned apartments will each have two habitable rooms that are 300 sf each. To determine the minimum operable area required per room, the area of the individual habitable space is multiplied by the minimum operable area percentage allowed:300 sf x 0.05 = 15 sf

A proposed two-story wood-framed building is 36'x48'. The architect is comparing costs for two different framing systems for the second floor. The first system is 18"-deep trusses spaced 24 inches on center. The cost of the trusses is $3.20 per linear foot. The second system is 16" wood I-joists spaced 16 inches on center. The cost of the I-joists is $2.50 per linear foot. The joists will span the short direction. Ignoring any cost differences due to increased building height and assuming all other factors are the same (including labor, fasteners, intermediate supports, and lateral bracing), what is the cost difference between the two framing systems for the second floor? Round to the nearest whole dollar.

Incorrect - the answer is 450 This is a unit cost comparison. First, figure out how many framing members are needed when they are spaced at 24 inches on center or 16 inches on center. Do this by sketching the building's footprint. The framing members will span in the short direction.48 ft x 12 in. / 24 in. = 24 + 1 = 2548 ft x 12 in. / 16 in. = 36 + 1 = 37 (An additional framing member has been added to each calculation above to accommodate the extra member required at one end of the building. This can been seen by sketching the framing members in the building's footprint.)The placement of trusses at 24 inches on center would require 25 members, and the placement of I-joists at 16 inches on center would require 37 members.Now calculate the price of each by taking the number of members x the cost per linear foot x the length (36 ft).Trusses:25 x 36 ft x $3.20 = $2,880.00I-joists:37 x 36 ft x $2.50 = $3,330.00Last, find the difference by subtracting. $3,330.00 - $2,880.00 = $450.00.

An architect is calculating the annual depreciation value for a geothermal system. The system costs $22,000 to install and has an expected useful life of 35 years. The system has a salvage value of $1,500 and requires maintenance every 5 years, at $500 per scheduled maintenance appointment. What is the annual depreciation value of the geothermal system?

Incorrect - the answer is 586 First, subtract the salvage value from the initial cost: $22,000 - $1,500 = $20,500 Then, divide by the useful life expectancy of the system: $20,500 / 35 years = $585.71, which rounds to $586. The maintenance costs are irrelevant to annual depreciation calculations.

During a low-grade seismic event, the two-hinge moment-resisting frame depicted undergoes elastic deformation. What is the moment value at point B if the two-hinged moment-resisting frame resists a seismic force of 70 kips? Ignore the weight of the frame itself.

Incorrect - the answer is 595 Because the two columns are identical (e.g., there are two HSS12x12 columns), each column resists ½ of the total seismic force; so we divide 70 kips / 2 = 35 kips (this force will act horizontally upon the hinged connection at the base of each column). The height of the columns is 17 ft. Thus, the moment value at point B will be:M(B) = 35 kips x 17 ft = 595 kip-ft.

During a low-grade seismic event, the two-hinge moment-resisting frame depicted undergoes elastic deformation. What is the moment value at point B if the two-hinged moment-resisting frame resists a seismic force of 70 kips? Ignore the weight of the frame itself.

Incorrect - the answer is 595 Because the two columns are identical (e.g., there are two HSS12x12 columns), each column resists ½ of the total seismic force; so we divide 70 kips / 2 = 35 kips (this force will act horizontally upon the hinged connection at the base of each column). The height of the columns is 17 ft.Thus, the moment value at point B will be:M(B) = 35 kips x 17 ft = 595 kip-ft.

A proposed children's play area contains the following equipment: 1 sandbox 9 swings 1 spiral standalone slide 3 play panels 1 dome climber 2 diggers 1 playhouse How many of the above play items must be on an accessible route?

Incorrect - the answer is 7 The answer is 7. Scoping for play structures is found in the ADA 2010 Standards section 240.2. One of each kind of ground-level play feature (sandbox, swing, standalone slide, play panels, climber, digger, and playhouse) must be on an accessible route. See how there is a different between "accessible route & ADA usable"

An architect is designing a new residential building for a client who prefers to meet the minimum code requirements and nothing more. Each residential unit will have a master bedroom that is 500 square feet and a second bedroom that is 250 square feet. What is the minimum area of glazing required for the two bedrooms combined?

Incorrect - the answer is 8640 Correct. IBC (2018) 1204.2 requires a minimum net glazed area of 8% of the floor area. The total area of the two rooms is 750 sq. ft., and 8% of that would be 60 sq. ft. (8,640 sq. in.). There are 144 square inches in 1 square foot. To convert from square feet to square inches, multiply your figure by 144

An architect is designing a building in a cold climate and wants to eliminate condensation on the interior surface of the building's exterior wall. Which of the following will help eliminate condensation?

Increase the wall insulation Thermal insulation will help to reduce the amount of heat lost from a building and raise the internal temperature of the wall near the interior surface. Lowering the interior relative humidity Lower humidity will result in a lower dew point for the air within the space - thus lowering the chance of condensation Raising the surface temperature of the affected area Increasing the surface temperature will reduce the cooling of any moisture-laden air and, consequently, the amount of condensation.

An architect is working on a new ski resort clubhouse. In an effort to keep the mechanical equipment on the roof hidden, the architect has decided to enclose the equipment with a large screen wall. The screen wall will be attached to the main roof structure at the corners with posts. Which of the following changes to building loads will have the greatest impact on the structure?

Increased snow load Correct. Snow drift occurs when vertical members are present on roofs; the snow drifts towards those members, and piles up against them. This also can occur at parapet walls on flat roof construction. The increased snow load due to drifting will increase the load on the roof framing. This increase can impact a large portion of the roof system depending on the overall size and geometry of the equipment screen. Increased roof dead load Incorrect. The increased roof dead load will be minimal as the load from the screen wall will be transferred to the main structure through each of the four posts, meaning that each post will apply a point load to the structure equal to 1/4 of the total screen wall load.

An architect is designing a building in a cold climate and wants to eliminate condensation on the interior surface of the building's exterior walls. Which of the following will help eliminate condensation?

Increasing the wall insulation Thermal insulation will help to reduce the amount of heat lost from a building and raise the internal temperature of the wall near the interior surface. Lowering the interior relative humidity Lower humidity will result in a lower dew point for the air within the space - thus lowering the chance of condensation. Raising the surface temperature of the affected area Increasing the surface temperature will reduce cooling of any moisture-laden air and, consequently, the amount of condensation. Topic: Project Integration of Program & Systems

It's wintertime, its been a rainy months and there moisture inside the parapet structure. The building included an arboretum. This can most likely best addressed with..... Insulation Rain barrier Vapor barrier Ventilation

Insulation By adding a continues barrier of insulation to remove the thermal bridge. By doing this you are eliminating the thermal bridge. Temperatures inside the wall cavity will remain the same as the inside allowing for better control of moisture (Bigger Glass)

An architect is working on a project in an unfamiliar location. Where should the architect look to determine the climate zone of the project site?

International Energy Conservation Code (IECC) Climate data for building science is found in the International Energy Conservation Code. These maps are used to determine the climate zone for designing building envelopes and mechanical systems in buildings. The USDA ZOne map is incorrect. This map is the standard by which gardeners and growers can determine which plants are more likely to thrive at a location.

An architect in the U.S. is working to develop standard procedures for code reviews in their office. To ensure consistency, they are looking to create a checklist of building codes to review on each project. In addition to the International Building Code, which of the following codes should always be reviewed regardless of project type?

International Energy Conservation Code (IECC) Correct. The International Energy Conservation Code (IECC) should be reviewed on every project to ensure energy performance requirements are met. NFPA 70 Correct. NFPA 70, more commonly referred to as the National Electric Code, should be reviewed on every project. ADA Standards for Accessible Design Correct. The Americans with Disabilities Act (ADA) Standards for Accessible Design should be reviewed on every project to determine what accessibility requirements may be necessary for the project. NFPA 101 Correct. NFPA 101, or the Life Safety Code, should be reviewed on every project. NFPA 13 Incorrect. NFPA 13 is only required when a sprinkler system is part of the project.

An architect is working on a renovation to an existing manufacturing plant. During the design, a portion of the manufacturing space needs to be changed from an F-1 occupancy to an H-1 due to the hazardous materials involved in the manufacturing process. As a result, the architect is performing another code review to ensure all code-required changes have been captured. Which of the following will need to be reviewed for changes due to the change in occupancy?

International Mechanical Code Correct. The International Mechanical Code should be reviewed to incorporate the necessary requirements for a high hazard occupancy. NFPA 13: Standard for the Installation of Fire Sprinkler Systems Correct. The NFPA 13 fire sprinkler standards should be reviewed again for any additional requirements due to the change from an F-1 to H-1 occupancy. Life Safety Code (NFPA 101) Correct. NFPA 101, the Life Safety Code, should be reviewed to incorporate any changes required by the change in occupancy. National Electric Code (NFPA 70) Correct. The National Electric Code, NFPA 70, should be reviewed to ensure the high hazard requirements are incorporated. Note: ICC A117.1-2009 Accessible and Usable Buildings and Facilities Incorrect. ICC A117.1 does not need to be reviewed again as the accessibility requirements will not change due to a change in occupancy classification of the space. If any of the physical characteristics of the space are changed, then accessibility standards should be re-reviewed.

An architect is designing a passive solar building which will take advantage of an adjacent south-facing slope. The slope will have a specially designed collector where air will be warmed by the sun and then rise to serve as the heat for the building. Cooler air will simultaneously be pulled from a chamber below the floor, creating a convection loop. How is this type of passive solar system identified?

Isolated gain Correct. Isolated gain systems collect and store solar radiation away from the space to be heated, as described. Isolated gain, or sunspace, passive heating collects the sunlight in an area that can be closed off from the rest of the building. The doors or windows between the sunspace and the building are opened during the day to circulate collected heat, and then closed at night, allowing the temperature in the sunspace to drop.

There are changes to the appliances planned for a new building. In order to accommodate the electrical needs of the new appliances, the voltage for the system must be doubled and the resistance cut in half. How do the changes affect the electrical current?

It is quadrupled Correct. Ohm's Law states that current is directly proportional to voltage and inversely proportional to resistance. State this as the equation I = V/R where I is current, V is voltage, and R is resistance. In this scenario, V increases by a factor of 2 and R decreases by a factor of 1/2. Solving for I, we find that I changes by a factor of 4. The electrical current is quadrupled.

The architect and structural engineer for a 73-story apartment building are evaluating the options for the structural framing system for the building. Because of the building's height, they need a system that will resist the lateral forces imposed on the structure. Which of the following framing systems would be appropriate for this high-rise structure? Check the four that apply.

Latticed truss Correct. A latticed truss has perimeter frames of closely spaced diagonals with no vertical columns, and is appropriate for high-rise buildings. Trussed tube Correct. A trussed tube has trussed wall frames with widely spaced columns that are tied together with either diagonal or cross bracing, making it appropriate for high-rise construction. Bundled tube Correct. A bundled tube structure ties narrow vertical tubes together to form a modular structure that is appropriate for high-rise buildings. Diagrid Correct. A diagrid is a diagonal grid frame where the diagonal members are connected at specially jointed nodes. This exterior framework has the potential for reducing the number of internal supports. It provides both shear and bending rigidity and is highly effective in resisting the lateral forces of a high-rise building. Rigid frame Incorrect. A rigid frame is only appropriate for low or mid-rise buildings, since it is the least efficient way to achieve lateral stability.

The sustainability goals for a new urban office building include cool roofing in order to reduce the heat island effect. Which of the following would be the most effective?

Light gray concrete tile Correct. This is the best option because this material has both a high reflectance and high emissivity. White asphalt shingles Incorrect. This material has high reflectance but low emissivity (it retains the heat that it does not reflect).

The sustainability goals for a new urban office building include cool roofing in order to reduce the heat island effect. Which of the following would be the most effective?

Light gray concrete tile Correct. This is the best option because this material has both high reflectance and high emissivity. Note:White asphalt shingles Incorrect. This material has high reflectance but low emissivity (it retains the heat that it does not reflect).

An architect is performing a code review for a new school building. During the review, the architect reviews the International Energy Conservation Code in addition to the International Building Code. Which of the following items are controlled by the International Energy Conservation Code (IECC)?

Lighting controls Correct. The IECC specifies lighting control requirements to maximize energy efficiency. Electrical power consumption Correct. The IECC specifies the maximum electrical power consumption for different features such as electric water heaters and lightin Wall insulation requirements Correct. The IECC specifies the wall insulation requirements. Total building performance requirements Correct. The IECC denotes the requirements for total building performance. Glazing thickness Incorrect. While the IECC specifies thermal performance criteria for glazing, the overall construction of the glazing assembly is controlled by the IBC.

For a community library that is being remodeled, the client has stated that all new materials must be as sustainable as possible and within the budget. After doing some research, the client asked the architect to use a flooring material consisting of linseed oil, ground limestone, and powdered wood for the staff break room. What type of flooring is being requested by the client?

Linoleum (lino) Correct. Many of the ingredients in linoleum are renewable.

An architect is calculating the allowable area of a building in accordance with the International Building Code. Which areas are included in the area calculation?

Lobby vestibule Correct. Per the International Building Code (2018), the area of a building is defined as "the area included within surrounding exterior walls (or exterior walls and fire walls) exclusive of vent shafts and courts. Areas of the building not provided with surrounding walls shall be included in the building if such areas are included within the horizontal projection of the roof or floor above". Storage rooms Exterior walls Incorrect. The building code's definition of the area of a building is taken from the inside face of the exterior wall. Mezzanine Incorrect. The floor area of a mezzanine is not included in the building area.

An architect is designing a custom residence on a large site that abuts a busy highway. Minimizing the impact of the highway noise is a critical part of the project design. Which of the following would be the best option for mitigating noise from the highway?

Locate the house as far from the highway as possible Correct. Noises will dissipate over distance. so the site plan should be designed to locate the house as far from the noise source as possible. Plant trees along the highway Incorrect. Trees and shrubs can slightly help reduce noise levels by absorbing some sound, but they will not significantly block the noise.

What is an Air-handling unit?

Located down the hall, cools air for delivery to the room via ductwork.

What is a Fan Coil Unit?

Located in the room, cools air by blowing it over pipes filled with chilled water from a chiller

A 120,000-sq. ft. high school building located in a mild climate, is being retrofit with a horizontal ground source heat pump. There is a limited amount of room on the site to place the piping. Which features should be incorporated into the system?

Loops or coils in the piping Correct. "Slinky" designs allow for more piping in less space. A hybrid system with multiple heating or cooling sources Incorrect. Hybrid systems are for climates with imbalanced heating and cooling needs.

What is the Evaporator?

Low pressure cold refrigerant

An architect is conducting code analyses for several projects and the process includes reviewing the egress requirements. Panic hardware on exit doors would be required for which of the following scenarios?

Main exit for a small private high school with 75 students Gates in the fence at an outdoor swimming pool serving 60 people Rear exit for a large restaurant serving 220 people Correct. Panic hardware is required for A and E occupancies with 50 or more occupants. Main door for a storage facility housing hazardous materials with an occupant load of 3 Correct. Panic hardware is required for high-hazard occupancies regardless of occupant load. Main exit for a preschool with 35 occupants Incorrect. The occupancy load is under 50, so panic hardware is not required.

An architect is conducting code analyses for several projects and the process includes reviewing the egress requirements. Panic hardware on exit doors would be required for which of the following scenarios?

Main exit for a small private high school with 75 students Correct. Panic hardware is required for A and E occupancies with 50 or more occupants. Main door for a storage facility housing hazardous materials with an occupant load of 3 Correct. Panic hardware is required for high-hazard occupancies regardless of occupant load. Gates in the fence at an outdoor swimming pool serving 60 people Correct. Panic hardware is required for A and E occupancies with 50 or more occupants. An outdoor pool is an A occupancy. Rear exit for a large restaurant serving 220 people Correct. Panic hardware is required for A and E occupancies with 50 or more occupants. 27-story apartment building with 8 apartments on each floor Incorrect. R occupancies do not require panic hardware.

An architect is creating a hotel design for a hospitality client with properties in four distinctly different climates. The client would like to use a single design for all the locations while utilizing natural light to reduce energy use. Which of the following daylighting strategies should the architect implement in all climate regions to improve energy efficiency? Check the four that apply.

Maintain on-site daylight access Correct. Arrangement of adjacent buildings, site features, and building orientation provides daylighting opportunities. Utilize reflection and incorporate diffused light to promote daylighting deeper into the building core Correct. Reflected and diffused light redirect daylight effectively, while reducing glare into a building's interior. Rooms must be well- proportioned and sized correctly for this strategy to work. Manage glare by specifying tinted glass Correct. Mitigating glare ensures occupants will refrain from using blinds to completely block daylight. Modify the landscape design to use planting to modulate and filter sunlight based on each location Correct. In a temperate climate in the northern hemisphere, the architect could use deciduous trees or vines to shade the south facade in winter, and allow light and radiation in the winter. In a cold climate in the northern hemisphere, the architect might avoid plantings along the south facade. In an arid climate in the northern hemisphere, the architect could specify drought-tolerant trees around the east, west and south facades to provide shade.

An architect is sizing downspouts and leaders for a multifamily project. Which of the following are determinants of minimum gutter or leader sizing?

Maximum hourly rainfall Correct. The maximum hourly rainfall for a specific location will be used to determine gutter and leader sizes. Roof area Correct. The square footage of a roof that contributes to a specific gutter or downspout will be used to determine sizes. Slope of pipe Correct. The slope of a gutter will be used to determine its required size. Smaller gutter slopes warrant larger channel sizing. Maximum annual rainfall Incorrect. The maximum amount of rainfall is not used to determine sizing of these elements. Rather, the maximum rate of rainfall for a location is used. Slope of roof Incorrect. The slope of the roof is not used to determine the sizing of gutters or leaders, but it would affect placement of these elements.

Should crawl spaces be ventilated or sealed?

New, evidence-based, thinking: Crawl spaces should be sealed: treat them like (small) basements and think of them as part of the building's thermal enclosure. Underlay them with gravel, plastic sheeting, and a concrete slab to keep out groundwater, insulate the walls (but not the overhead plane), and heat/cool them just like the house.

An architect is sizing downspouts and leaders for a multifamily project. Which of the following are determinants of minimum gutter or leader sizing?

Maximum hourly rainfall Correct. The maximum hourly rainfall for a specific location will be used to determine gutter and leader sizes. Slope of pipe Correct. The slope of a gutter will be used to determine its required size. Smaller gutter slopes warrant larger channel sizing. Roof area Correct. The square footage of a roof that contributes to a specific gutter or downspout will be used to determine sizes Slope of roof Incorrect. The slope of the roof is not used to determine the sizing of gutters or leaders, but it would affect placement of these elements. Maximum annual rainfall Incorrect. The maximum amount of rainfall is not used to determine sizing of these elements. Rather, the maximum rate of rainfall for a location is used.

An architect is surveying a 4,500-square-foot cabin in the northwestern United States that is slated for renovation. After bringing in a structural engineer, it is determined that the existing shallow foundations are not capable of bearing the existing loads and the renovation will require deep foundations. What foundation type is best suited for this situation?

Micropiles Correct. Since this is a renovation project and the existing structure is to remain, the smallest and least invasive foundations should be used. Micropiles are installed through the downward force provided by a hydraulic jack. A hydraulic jack is small enough to get close to the existing structure to drive the micropiles in at an appropriate location. H-piles Incorrect. The installation of H-piles requires a pile driver. A pile driver is a large and cumbersome piece of equipment, and it is unlikely that it could get close enough to the existing structure to locate the piles in a place where they could support the existing load.

Piles for a new building's foundation must be used in a sensitive area adjacent to another building. Vibration and disturbance must be kept to a minimum. Which type of pile could be used?

Mini Correct. Minipiles (or micropiles) are small high-strength piles. A small steel casing is drilled into the ground. Steel bars are then inserted into the casing and grouted in place. The smaller size of minipiles makes them suitable for installations where minimizing impact is a main criterion. Helical Correct. Helical piles screw into the soil like a nail screws into wood. This means there is no soil displacement, making them suitable for installations where minimizing impact is a main criterion. Sheet Incorrect. Sheet piles are for excavations, not foundations.

A developer is evaluating alternative sites for a luxury residential development. Despite higher development costs, they feel that a hillside site will warrant greater market values because of the views and unique character of the terrain. What is typically the most cost-effective means to develop the site while retaining its character?

Minimize disturbed areas Correct. Minimizing disturbed areas on the selected site is typically more cost-effective and will have less of an impact on natural drainage patterns and habitats. It also allows the finished grading to more closely mimic the original terrain and retain the character of the site. Straighten the slopes Incorrect. Straightening the existing slopes on the selected site can negatively impact the natural appearance of the terrain and affect drainage patterns.

An architect is interested in incorporating a vegetated roof on an existing office building. The system needs to be as light as possible to minimize additional loading on the existing structure. Which of the following vegetated roof systems should the architect propose?

Modular block vegetated roofing system Correct. The modular block system typically only require 3-4 inches of engineered soil to sustain growth. This typically adds an average of 12-18 pounds per square foot of additional load to the structure. Extensive vegetated roofing system Incorrect. An extensive system typically requires 4-6 inches of engineered soil and can add roughly 15-50 pounds per square foot to the roof structure. A modular block system is more appropriate.

An architect is interested in incorporating a vegetated roof on an existing office building. The system needs to be as light as possible to minimize additional loading on the existing structure. Which of the following vegetated roof systems should the architect propose?

Modular block vegetated roofing system Correct. The modular block system typically only require 3-4 inches of engineered soil to sustain growth. This typically adds an average of 12-18 pounds per square foot of additional load to the structure. Extensive vegetated roofing system Incorrect. An extensive system typically requires 4-6 inches of engineered soil and can add roughly 15-50 pounds per square foot to the roof structure. A modular block system is more appropriate. Intensive vegetated roofing system Incorrect. Intensive vegetated roofing systems require a minimum of 12 inches of soil and can add anywhere between 80 and 150 pounds per square foot to the existing structure.

The architect for a new forestry station in a national park has ordered a geotechnical survey with six soil borings to confirm the soil conditions of the project site. They will use the recommendations from the survey to determine the type of foundation needed to support the building. What types of soil information will the geotechnical survey provide? Check the four that apply.

Moisture content Correct. Soil borings will indicate the amount of moisture that is being held by the soil. Cohesion Correct. Soil borings will indicate the cohesion of the soil, which is the strength of the soil when unconfined and air-dried. Permeability Correct. Soil borings will test the soil's permeability, which is the amount of liquid or gas that can pass through its pore spaces. Shearing strength Correct. Soil borings will indicate the shearing strength of the soil, which is the ability of the soil to resist displacement from adjacent soil when an external force is applied to it. Frost level Incorrect. The frost level is not part of a soil report, but is a predetermined depth of frozen subsoil for a particular region of the country.

Refer to the plans of buildings A and B in the graphic. A building's planned columns are redistributed from the configuration in Figure A to now be on the periphery, as is shown in Figure B. What has increased as a result of this change?

Moment of inertia Correct. Moving mass farther from the centroid of either a beam or, in the case of this question, the whole building, increases the moment of inertia. Moment of inertia is defined as "the sum of the products of all the elementary areas times the square of their distances from the neutral axis." Moving the columns to the exterior of the building makes the structure stiffer as a whole. This is the same concept that gives the wide flange beam the "I-beam" shape.

How to calculate Square feet into Square inches?

Multiple the are value by 144 (12x12)

An architect in the U.S. is working to develop standard procedures for code reviews in their office. To ensure consistency, they are looking to create a checklist of building codes to review on each project. In addition to the International Building Code, which of the following codes should always be reviewed regardless of project type?

NFPA 101 Correct. NFPA 101, or the Life Safety Code, should be reviewed on every project. NFPA 70 Correct. NFPA 70, more commonly referred to as the National Electric Code, should be reviewed on every project. ADA Standards for Accessible Design Correct. The Americans with Disabilities Act (ADA) Standards for Accessible Design should be reviewed on every project to determine what accessibility requirements may be necessary for the project. International Energy Conservation Code (IECC) Correct. The International Energy Conservation Code (IECC) should be reviewed on every project to ensure energy performance requirements are met. NFPA 13 Incorrect. NFPA 13 is only required when a sprinkler system is part of the project.

An architect in the U.S. is working to develop standard procedures for code reviews in their office. To ensure consistency, they are looking to create a checklist of building codes to review on each project. In addition to the International Building Code, which of the following codes should always be reviewed regardless of project type?

NFPA 70 Correct. NFPA 70, more commonly referred to as the National Electric Code, should be reviewed on every project. NFPA 101 Correct. NFPA 101, or the Life Safety Code, should be reviewed on every project. International Energy Conservation Code (IECC) Correct. The International Energy Conservation Code (IECC) should be reviewed on every project to ensure energy performance requirements are met. ADA Standards for Accessible Design Correct. The Americans with Disabilities Act (ADA) Standards for Accessible Design should be reviewed on every project to determine what accessibility requirements may be necessary for the project. NFPA 13 Incorrect. NFPA 13 is only required when a sprinkler system is part of the project.

A client has asked an architect to design a loft with exposed brick in the interior. What grade of brick should the architect specify?

NW Correct. NW stands for negligible weathering. Grade NW is the least durable and is typically used only for interior applications.

A client has asked an architect to design a loft with exposed brick in the interior. What grade of brick should the architect specify?

NW Correct. NW stands for negligible weathering. Grade NW is the least durable and is typically used only for interior applications. SW Incorrect. SW stands for severe weathering. Grade SW is used in exterior applications where there are more severe weathering conditions. MW Incorrect. MW stands for moderate weathering. Grade MW units are less durable and are typically used for face brick in parts of the country where the weathering index is not as severe. FBX Incorrect. FBX means the bricks have minimum variation in size and color and have a high level of uniformity. The FBX designation has nothing to do with durability.

A client has asked an architect to design a loft with exposed brick in the interior. What grade of brick should the architect specify?

NW Correct. NW stands for negligible weathering. Grade NW is the least durable and is typically used only for interior applications. MW Incorrect. MW stands for moderate weathering. Grade MW units are less durable and are typically used for face brick in parts of the country where the weathering index is not as sev

An architect is working on a renovation to an existing manufacturing plant. During the design, a portion of the manufacturing space needs to be changed from an F-1 occupancy to an H-1 due to the hazardous materials involved in the manufacturing process. As a result, the architect is performing another code review to ensure all code-required changes have been captured. Which of the following will need to be reviewed for changes due to the change in occupancy?

National Electric Code (NFPA 70) Correct. The National Electric Code, NFPA 70, should be reviewed to ensure the high hazard requirements are incorporated. NFPA 13: Standard for the Installation of Fire Sprinkler Systems Correct. The NFPA 13 fire sprinkler standards should be reviewed again for any additional requirements due to the change from an F-1 to H-1 occupancy. International Mechanical Code Correct. The International Mechanical Code should be reviewed to incorporate the necessary requirements for a high-hazard occupancy. Life Safety Code (NFPA 101) Correct. NFPA 101, the Life Safety Code, should be reviewed to incorporate any changes required by the change in occupancy. ICC A117.1-2009 Accessible and Usable Buildings and Facilities Incorrect. ICC A117.1 does not need to be reviewed again as the accessibility requirements will not change due to a change in the occupancy classification of the space. If any of the physical characteristics of the space are changed, then accessibility standards should be re-reviewed.

The architect and owner of a proposed new neighborhood cultural center are seeking input from stakeholders with an interest in the project. In which of the following groups would the stakeholders represent their own individual interests?

Neighborhood business owners Correct. Individual neighborhood business owners can speak for themselves and represent the "typical" views of other business owners in their neighborhood. Neighborhood associations Incorrect. Neighborhood associations would fit into the "interest group" category of stakeholders. Individuals in these associations represent their organization's interests rather than their own individual interests.

A structural engineer is designing for the loads of several multi-story buildings that have different uses. For which of the following buildings should the engineering design for a partition load?

Office buildings In buildings where partition locations are likely to change, such as office buildings, provisions for a partition load of 15 psf shall be made, if the live load is less than 80 psf. See International Building Code (IBC, 2018) Section 1607.5.

An architect is reviewing product information for fire suppression systems. The product info states that ordinary sprinkler heads operate between 135 and 170 F, whereas the newer quick- response heads trigger at a lower temperature. Which of the following building types should the architect specify quick-response sprinklers for? Check the four that apply.

Offices Multi-family residential Hotels Assisted living facilities Correct. All light hazard occupancies are required to have quick-response (or fast-response) sprinklers. These include hotels, motels, offices, residential, healthcare, and other buildings where faster sprinkler operation could enhance life safety. The biggest difference between quick-response and standard-response sprinkler heads is their activation temperatures. As the name suggests, quick-response sprinkler heads activate sooner. Factory Incorrect. Standard-response sprinklers are used in factories and warehouses, where occupancy is low.

Where should I locate a ¨vapor barrier¨

On the warm side of the insulation, right up against the insulation. For a warm climates that means place the vapor barrier on the outside face of the insulation. For a cold climate that means position the vapor barrier on the inside face of the insulation.

An architect is specifying a fire suppression system for a small museum building that will house sensitive and expensive artifacts. Which type of sprinkler should the architect specify?

Preaction System A preaction system must go through a two-step process before the sprinklers will realize water. This system is designed to protect spaces with sensitive and expensive materials. The main advantage of pre-action systems is found in the two steps needed for water to release. Since both the pre-action valve and the individual sprinkler head must activate in order for water to be released, it minimizes the possibility of accidental discharge.

Put these in order from most effective insulation to least effective insulation

Perlite-Vermiculite-Most insulation. Perlite and Vermiculite are puffy rocks that have air pockets for thermal resistance, and because they are rocks, they can be exposed to moisture without significantly degrading. Air: More insulation than grout! Air molecules sit farther apart from one another than grout molecules, so air conducts heat more poorly than grout (Which is a good thing when using empty cavities in cold climates) Grout: Pourable and cementitious, grout serves as surprisingly poor insulation, until remember that all dense cementitious serve as poor insulation.

An architect is working on a renovation to an existing office building. The owner has asked to incorporate sustainable practices on the property, but would like to minimize the impact to the existing building as much as possible. Which of the following solutions should the architect suggest?

Photovoltaic array Correct. Adding a photovoltaic (PV) solar array would be the best suggestion given the desire to minimize the impact on the existing building. PV arrays can be ground-mounted, which eliminates the need to attach to the roof, and the connections can be made within the electrical room, meaning that the only modifications required are the new inverter and battery bank. Geothermal heat Incorrect. A geothermal heating system would involve substantial reconfiguration of the existing heating and cooling system, as well as additional disruption to the facility during the drilling of the wells required for the system to operate.

In the site investigation for a townhome development, the geotechnical report describes the building site as four feet of organic fill over twenty-four feet of silty sand on top of bedrock. Which of the following systems would be the best option for the townhomes?

Pile foundation Correct. A pile foundation is a type of deep foundation which is used to transfer heavy loads from the structure to bedrock. In this instance, the bedrock is not terribly far below the surface, and thus pile foundations are a viable option. Combined footings Incorrect. A combined footing is constructed when two or more columns are close enough that their footings would overlap each other. This is used for isolated instances and is not a type of foundation system. Spread footings Incorrect. Spread footings are footings whose base is wider than a typical load-bearing wall foundation. This is the typical option for smaller structures on stable soil.

An architect is preparing the design and detailing for a below-grade basement for a new custom home. The home sits on a site with a variable water table where moisture intrusion and hydrostatic pressure are of concern. Which of the following are the most effective methods for creating a waterproof basement? Check the three that apply.

Placing a moisture barrier before pouring the foundation Correct. Placing a moisture barrier below the foundation will aid in reducing water and moisture penetration up into the slab. Placing a course of gravel beneath the foundation pour Correct. The gravel course will provide a porous surface for water to drain into the ground below as opposed to soil, which may gather water and cause capillary or increased hydrostatic pressure on the foundation slab. Placing perforated drain tile around the foundation, above the concrete footing Correct. Placing the pipe around the foundation will allow water to flow away from the foundation to keep the basement dry. Applying damp proofing around the exterior foundation wall Incorrect. Dampproofing is a layer applied during the foundation process, intended to keep soil moisture out of the building. It is not an effective long-term solution, as it is unable to seal large cracks in the concrete and may be subjected to damage during backfill operations.

The owner and architecture team are developing a property into a country estate and golf course. They are looking at wetlands as a sustainable strategy for managing stormwater. The area available for wetlands development is approximately six acres, shoe-horned between the country estate and the golf course. As an active course, the owner would like to minimize the attraction of native species and wildlife. Overall, the project is aiming for a moderate pollutant removal in coordination with other treatment measures onsite. Which one of the following wetland designs would the civil engineer most likely recommend?

Pocket wetland Correct. The pocket wetland is suitable for up to 10 acres and due to the smaller size, is not an attractant for wildlife. In addition, it has only moderate pollutant removal, and is compatible with the stated goals of the project. Also, the system is more malleable to fit into the ancillary space between the estate and the golf course. Shallow marsh Incorrect. This system would require approx 25 or more acres of available land to be effective, as shallow marsh consumes much land. In addition, the large spread would attract wildlife. This system is more applicable to high pollutant removal, which is not necessary for the stated goals of this wetland project. Pond wetland Incorrect. This system would require approx 25 or more acres of available land to be effective. This system also tends to attract waterfowl and cleans a very high level of pollutants, which is not compatible with the programming goals of this project. Detention pond Incorrect. Requires a minimum of 10 acres to be effective. While the removal of pollutants would be in line with the goals, it also tends to attract wildlife due to the size of the pond, and is not compatible with programming goals of this project.

An architect is working on a new high school building. The initial cost estimate determines that the project is over budget, and it is recommended that value engineering be performed to get the project's budget back on track. Which of the following are reviewed during value engineering exercises?

Product initial costs Correct. Product initial costs are evaluated during value engineering. A higher initial cost can sometimes be justified if cost savings can be accumulated through lower installation costs and longer life expectancy. Energy usage cost Correct. By evaluating the energy usage cost for equipment, it can be determined if a higher initial cost can be justified due to cost savings over the lifespan of the product. Replacement costs Correct. Replacement costs are evaluated during value engineering as a product may have a lower initial cost but higher replacement costs. Depending on the product, it may be determined that it is more economical to provide a product with a higher initial cost and lower replacement cost, or vice versa. Life expectancy Correct. The life expectancy of a product is reviewed during value engineering in order to evaluate its total cost over its entire lifespan. Local availability Incorrect. While selecting products that are available locally does have its benefits, this is not a factor that is typically reviewed during value engineering.

The first modern zoning code was introduced in New York City in 1916. Which of the following exemplify the purpose of zoning?

Protect access to direct sunlight and fresh air Correct. The need for zoning began with the urban growth in the late 19th- early 20th century when it became clear that cities such as New York would soon have streets with limited access to light and air. Ensure incompatible types of building uses do not disturb residents Correct. Zoning is centered on protecting the rights of residents and property owners. This includes disturbances such as putting a school next to a noisy factory. Protect local residents from undesirable types of businesses Correct. Protecting residents from undesirable businesses is a key component of zoning. This concept is in place in order to group compatible businesses and ensure the wrong kinds of businesses don't end up too close to each other. Protect the amount of open space for local residents Correct. The amount of open space directly correlates to the need to maintain access to light and air. Determine the allowable function of a building Incorrect. The allowable function of a building falls under building rather than zoning code. Building codes comprise a set of rules that specify minimal quality and safety requirements. Regulate the number of people or occupant loads of a building or structure Incorrect. Occupancy falls under building rather than zoning code. Building codes comprise a set of rules that specify minimal quality and safety requirements.

The owner provides a topographic survey of the greenfield site of a new shopping mall. The architect noted the site has several steep vegetated hills. What site strategies should be put in place to mitigate erosion?

Protect and retain existing vegetation Correct. Existing vegetation is already established on the hillside and helps prevent erosion from rainwater. Design development to fit the site and the terrain Correct. Situating the building on an area of the site with flat terrain will limit disturbance of existing soils, which leads to a greater chance of erosion. Revegetate and mulch exposed areas Correct. Added vegetation and mulch help to hold disturbed soil in place and limit erosion from runoff. Strip topsoil from steep hills Incorrect. The loss of topsoil creates major site damage. The onsite loss of this upper layer of soil nearly eliminates the soil's natural ability to provide nutrients, regulate water flow, and combat pests and disease.

Fire protection is an important part of the design of a new building. Which of the following choices accurately lists the priorities for designing for fire protection in buildings?

Protecting the building's occupants > Protecting the building itself > Protecting the building's contents Correct. Protecting the people inside the building is the top priority, followed by protecting the building, and then the things inside the building. Ensuring access for fire and rescue personnel > Providing adequate water supply for fire-fighting > Limiting quantity of combustible material Incorrect. These are not the guiding priorities (which are protection of the occupants, the building, and the contents), but they are important details that will affect the protection of the priorities. Therefore, how rescue personnel access the site and combat the fire event is addressed throughout various sections of the IBC and municipal code.

An architect is designing a five-home townhouse development where each townhouse will have a full basement. The geotechnical engineer advised the team that groundwater is expected to be found 5 to 6 feet below grade. Which of the following should be included in the design in order to mitigate the effects of groundwater on the foundation?

Provide a foundation drain Correct. Foundation drains allow groundwater to be mitigated and diverted away from the foundation walls.

An architect is designing a five-home townhouse development where each townhouse will have a full basement. The geotechnical engineer advised the team that groundwater is expected to be found 5 to 6 feet below grade. Which of the following should be included in the design in order to mitigate the effects of groundwater on the foundation?

Provide a foundation drain Correct. Foundation drains allow groundwater to be mitigated and diverted away from the foundation walls. Provide joint sealant at the joint between the foundation slab and walls Incorrect. This should be provided to allow for expansion and contraction between the wall and slab of the foundation, as well as settling. However, this detail is not useful in mitigating the effects of groundwater on a foundation.

What is a variable refrigerant flow (VRF) HVAC system?

Pumps and manifolds move refrigerant to spaces that need heating (high-pressure refrigerant gets hot). And pumps and manifolds move refrigerant from spaces that need cooling (low-pressure refrigerant gets cold). An in-unit fan blows air over the refrigerant coil in each space. Because we can simultaneously heat the perimeter while cooling the core-just by moving refrigerant between the two-this type of system cab often be your lowest energy option. The ductless mini-split system we covered is a type of VRF.

The architects for a proposed recreation center building in an urban neighborhood are discussing the best options for receiving community input on the project. Which of the following options are ways to engage local community stakeholders in the design process? Check the three that apply.

Questionnaires Correct. Well-formatted questionnaires for local community members can solicit input on the needs and concerns of neighborhood stakeholders. Interactive workshops Correct. Interactive workshops with local community members allow for a give-and-take of ideas that are important to the local stakeholders and helpful to the architect. They can alleviate stakeholder concerns about the project and allow for understanding and support from the community. Design charrettes Correct. Engaging community members in design charrettes can be an effective and satisfying experience for local stakeholders while providing useful design input for the architect. Planning commission meetings Incorrect. Planning commission meetings are geared more toward comments and approvals from the planning board, rather than community members. Although community members often have an opportunity to make comments, it is not a proper forum for engaged community input.

A homeowner in a cold weather climate is remodeling their primary bathroom. Following their architect's suggestion, they will be installing an electric underfloor heating system under the tile flooring for increased thermal comfort in the winter. What is the main method of heat transfer that is utilized by an electric underfloor heating system to heat the air in the room?

Radiation Correct. The electric underfloor coils heat the floor by conduction. However, the air in the room is heated by thermal radiation from the floor. Conduction Incorrect. Conduction is the transfer of heat through a solid or liquid. In the case of an underfloor heating system, the floor itself is heated by conduction, but the room is heated by radiation from the floor.

A homeowner in a cold weather climate is remodeling their primary bathroom. Following their architect's suggestion, they will be installing an electric underfloor heating system under the tile flooring for increased thermal comfort in the winter. What is the main method of heat transfer that is utilized by an electric underfloor heating system to heat the air in the room?

Radiation Correct. The electric underfloor coils heat the floor by conduction. However, the air in the room is heated by thermal radiation from the floor. Convection Incorrect. Convection is the movement of heat through air or water. An example of this would be a forced air furnace system, where the air is heated by the furnace and physically blown through the space.

What is the difference between Restoration and Rehabilitation and Reconstruction?

Restoration is defined as "The act or process of accurately depicting the form, features, and character of a property as it appeared at a particular period of time by means of the removal of features from other periods in its history and reconstruction of missing features from the restoration period. Rehabilitation emphasizes the retention and repair of historical structures while also allowing latitude for repairs and replacement materials or building additions. Reconstruction Defined as "The act or process of depicting, by means of new construction, the form, features, and detailing of a non-surviving site, landscape, building, structure, or object for the purpose of replicating items appearance at a specific period of time and in its historic location"

A client is interviewing architectural firms regarding their experience and knowledge of evolving sustainable design strategies and technologies. Which strategies or technologies should the firm present to the client during the interview?

Regenerative design Correct. The goal of regenerative design is to produce a net positive environmental impact with respect to energy, water, and materials. This design strategy is new and evolving. Systems include biologically engineered wastewater treatment using living plant species to treat water, generate biomass in aquacultural ponds, promoting green roofs for stormwater management, carbon sequestration, and fauna habitat. Building Integrated Photovoltaics (BIPV) Correct. BIPV is an evolving solar energy technology that architects should be familiar with. The technology embeds photo-voltaic solar cells in the glazing itself, thus maximizing the surface area of buildings for electricity generation. Passive solar design Incorrect. Passive solar design has been around for centuries. The Greeks are considered to be among the first civilizations to design buildings for passive heating potential. Many of the same passive design strategies can be adopted in current design strategies, including building orientation, thermal massing, water harvesting, solar gain, and natural ventil USGBC (U.S. Green Building Council) LEED certification Incorrect. The USGBC LEED certification rating systems for green building design was launched in 2000 and is a popular certification worldwide. In this regard, this topic is not new and has been adopted widely by many U.S. cities into their building requirements. Most architect firms have LEED-accredited professionals on staff.

A three-story building housing a mineral spa was built in the 1920s as a tourist health destination. The building fell into disrepair in the 1960s and was unused for several decades. The building was listed on the National Register of Historic Places in 1979. In 2002, the first floor of the building was flooded when the nearby creek overfilled. The flooding left the building derelict and close to being condemned. A new owner has purchased the building and would like to convert it back to the mineral spa. Which treatment of historic structures should the architect recommend?

Rehabilitation Correct. Rehabilitation emphasizes the retention and repair of historical structures while also allowing latitude for repairs and replacement materials or building additions. By rehabilitating the space, repairs and appropriate replacements can be made while holding true to the original intent of the structure.

A three-story building housing a mineral spa was built in the 1920s as a tourist health destination. The building fell into disrepair in the 1960s and was unused for several decades. The building was listed on the National Register of Historic Places in 1979. In 2002, the first floor of the building was flooded when the nearby creek overfilled. The flooding left the building derelict and close to being condemned. A new owner has purchased the building and would like to convert it back to a mineral spa. Which treatment of historic structures should the architect recommend?

Rehabilitation Correct. Rehabilitation emphasizes the retention and repair of historical structures while also allowing latitude for repairs and replacement materials or building additions. By rehabilitating the space, repairs and appropriate replacements can be made while holding true to the original intent of the structure. Restoration Incorrect. Restoration is defined as "the act or process of accurately depicting the form, features, and character of a property as it appeared at a particular period of time by means of the removal of features from other periods in its history and reconstruction of missing features from the restoration period." There is no indication of the need to remove features from other periods of time. The building only needs to be rehabilitated.

An architect is developing the code review documents for a new medical facility. While performing the code review, the architect notices that certain types of medical facilities are classified as an Institutional occupancy and will require additional considerations. Which of the following medical facilities would be considered an Institutional occupancy?

Rehabilitation facility Correct. A rehabilitation facility is classified as an I-1 Institutional occupancy. Assisted living facility Correct. An assisted living facility is an I-1 Institutional occupancy. Hospitals Correct. A hospital is classified as an I-2 Institutional occupancy. Outpatient clinics Incorrect. An outpatient clinic is classified as a Business (B) occupancy.

The impervious area of the project site is 73 percent. Local code now requires a minimum of 50 percent pervious surfaces to minimize stormwater runoff and combined sewer overflow issues. Which solutions should the architect explore?

Replace portions of the parking lot with pervious pavement Correct. Adding pervious pavement surfaces in the parking lot will reduce stormwater runoff. Install underground stormwater storage below the parking lot Correct. Installation of underground stormwater collection and storage tanks beneath the parking lot is an accepted solution, especially for a site with 73 percent impervious surface areas that are mostly for parking. Install a green roof Correct. A green roof installation is an accepted solution for reducing stormwater runoff. It also adds insulation to the roof, sequesters carbon, and filters air pollution. Plant trees Correct. Planting trees will help collect and store rainfall in addition to providing shade (reducing heat island effect), and improving guest experience. Reduce parking lot surface Incorrect. The building classification code will determine parking lot size, so reduction of spaces may not be an option.

An architect is working on a new high school building. The initial cost estimate determines that the project is over budget, and it is recommended that value engineering be performed to get the project's budget back on track. Which of the following are reviewed during value engineering exercises?

Replacement costs Replacement costs are evaluated during value engineering as a product may have a lower initial cost but higher replacement costs. Depending on the product, it may be determined that it is more economical to provide a product with a higher initial cost and lower replacement cost, or vice versa. Life expectancy Energy usage cost By evaluating the energy usage cost for equipment, it can be determined if a higher initial cost can be justified due to cost savings over the lifespan of the product. Product initial costs Product initial costs are evaluated during value engineering. A higher initial cost can sometimes be justified if cost savings can be accumulated through lower installation costs and longer life expectancy. Note: While selecting products that are available locally does have its benefits, this is not a factor that is typically reviewed during value engineering.

An architect is designing a five-story addition to an existing building, and a separate foundation needs to be designed to support the addition. A subsurface investigation reveals that the soil on the site is predominantly coarse grained. Which type of foundation is most appropriate for the new building?

Shallow foundation with cantilever footings Coarse-grained soil on the site indicates that a deep foundation is not required. A cantilever footing addresses the need to build against an existing building.

An architect is designing a five-story addition to an existing building, and a separate foundation needs to be designed to support the addition. A subsurface investigation reveals that the soil on the site is predominantly coarse-grained. Which type of foundation is most appropriate for the new building?

Shallow foundation with cantilever footings. Coarse-grained soil on the sire indicated that a deep foundation is not required. A cantilever footing addresses the need to build against an existing building.

The architect selected the beam's size based on resistance to bending moment. What other properties of the beam should be checked before making the final determination on the beam size?

Shear, Deflection & Shear A wood beam is often first designed for file use (bending) and then checked for shear (specially horizontal) and deflection. The end bearing and lateral buckling are also checked. (Beams are note compression members)

An architect is working with a structural engineer on the design of a 1-story masonry retail building. Which of the following describes this building's likely fundamental period?

Short fundamental period Correct. The shorter and stiffer a structure or object, the shorter its fundamental period.

An architect has been commissioned to design an office that utilizes sustainable energy as much as possible. The office will be located in Arizona, so passive cooling will be an essential part of the design. Which of the following options would be the most effective at reducing the solar heat gain?

Shutters on the outside Correct. Reducing solar heat gain is best done by not allowing the sun's rays to enter the building. Exterior shutters stop the sun's rays by shading the glass. Light colored finishes inside Incorrect. Light colored finishes help exterior elements as they reflect the heat, but once solar heat has entered a building the interior finishes don't have a noticeable effect.

A building design is to include photovoltaic (PV) panels as a component of the electrical system. Which of the following is an appropriate material for photovoltaic cells?

Silicon Correct. Photovoltaic cells require a semiconductor to create the flow of electricity. Silicon is the only semiconductor of the materials listed here, though alternatives such as amorphous materials are also available.

An architect is preparing a cost of work estimate at the end of the schematic design phase for a 55,000-square-foot retail project. After studying past projects completed in their office, the architect has decided that the most similar project for which they have cost information is a different office completed three years ago for $250 per square foot. The owner has also provided the following information to be included in the architect's cost of work estimate: Design contingency: 5% Bidding contingency: 7% Construction contingency: 10% Finally, the owner asked the architect to factor in an escalation factor of 2%. Construction is expected to begin in one year. What is the total cost of work estimate? Round to the nearest thousand.

Since the office building that will be used as the basis for cost was completed three years ago and the scenario notes a 2% escalation, the first step is to determine the appropriate cost per square foot to price the project today: $250/sf x 1.02 x 1.02 x 1.02 = $265.30/sf ($265.302)(Note that numbers provided in parentheses show the math that would result from not rounding this first step. Either method is considered correct.)Next, multiply that cost by the square footage of the project: 55,000 sf x $265.30/sf = $14,591,500 ($14,591,610)Next, find the three contingencies noted in the scenario: Design contingency: 5% x $14,591,500 = $729,575 ($729,580.50) Bidding contingency: 7% x $14,591,500 = $1,021,405 ($1,021,412.70) Construction contingency: 10% x $14,591,500 = $1,459,150 ($1,459,161.00) Next, add these three contingencies to the base cost:$729,575 + $1,021,405 + $1,459,150 + $14,591,500 = $17,801,630 ($17,801,764.20)Last, apply the escalation factor to include the fact that construction is expected to begin in one year: $17,801,630 x 1.02 = $18,157,662.60 (18,157,799.48) Round the answer to the nearest thousand, which is $18,158,000.

SOH CAH TOA stands for?

Sine = Opposite/Hypotenuse Cosine = Adjacent/Hypotenuse Tangent = Opposite/Adjacent

When conducting a code analysis for a new apartment building, the project lead notes that the project's local jurisdiction has adopted the International Energy Code (IECC), limiting the total power that can be used for lighting a building. Which method may the architect and/or the electrical engineer employe in order to compy with the requirements?

Space by Space Method This method assigns allowable wattage per square foot to spaces based on their user type. When multiplied by the gross area of each programmatic space, a lighting power density can be determined and compared to the code requirement. Note that a GROSS AREA CALCULATIONS approach is incorrect. This measure accounts for exterior walls, including vertical penetrations, and does not consider energy usage. so it is not sufficient by used for determining whether the code has been met.

An architect is looking to implement more environmental strategies in the firm's upcoming projects. Which of the following building and site configurations is best suited to implement passive heating and cooling?

Small single-story residential building on a rural site Correct. Passive heating and cooling strategies are best implemented for small and medium-sized buildings that are on sites which provide flexibility in orientation and configuration. Large single-story office building on rural site Incorrect. Large buildings are not well suited for passive heating and cooling strategies because the typical configurations for these buildings are not compatible with the design elements that are necessary for passive design.

A surface with an absorption coefficient of 1.00 is considered _____ Sound - reflective or Sound - absorptive

Sound Absorptive Ranges from 0.0 (Fully sound reflective) to 1.0 (Fully Sound absorptive)

An architect is putting the finishing touches on the site plan for a new sports complex when an email comes in from the electrical engineer. It states that the kVA equipment has to be moved to the south end of the site. Which piece of electrical equipment carries a kVA rating?

Transformer Correct. Transformers and generators carry kVA ratings, which is a measure of power. kVA stands for kilovolt-amperes. Kilovolt-amperes are calculated by multiplying the volts and the amperes.

When conducting a code analysis for a new apartment building, the project lead notes that the project's local jurisdiction has adopted the International Energy Conservation Code (IECC), limiting the total power that can be used for lighting a building. Which method may the architect and/or the electrical engineer employ in order to comply with the requirements?

Space-by-space method Correct. The space-by-space method assigns allowable wattage per square foot to spaces based on their use type. When multiplied by the gross area of each programmatic space, a lighting power density can be determined and compared to the code requirements. Energy use calculations Incorrect. Calculating the energy use cumulates all the energy used for the operation of the building. Calculating energy consumption helps to determine if the building meets the code requirements, but it does not consider the program or square footage of the building, which is necessary for determining the total power that can be used.

When conducting a code analysis for a new apartment building, the project lead notes that the project's local jurisdiction has adopted the International Energy Conservation Code(IECC), limiting the total power that can be used for lighting a building. Which method may the architect and/or electrical engineer employ in order to comply with the requirement?

Space-by-space method This method assigns allowable wattage per square foot to spaces based on their user type. When multiplied by the gross area o each programmatic space, a lighting power density can be determined and compared to the code requirements. Note: "Gross are calculations" measure the area of all building floors within the exterior walls, including all vertical penetration areas of circulation and shafts connecting one floor to another. This measurement is necessary for calculating energy performance but does not consider energy, so it is not sufficient by itself to determine whether the code has been met.

The architect has chosen to utilize a criteria matrix in order to begin organizing the program. Which of the following spatial qualities would be listed in the criteria matrix?

Square footage Correct. Square footage of each program is typically listed in the criteria matrix to determine where it will fit. Adjacencies Correct. Adjacencies are typically listed so that similar and complimentary programs can be placed in proximity to each other. Public access Correct. Public access is typically listed to ensure both ease of access to the public programmatic requirements as well as privacy and security to other spaces. Daylight and view Correct. Both daylight and view are typically listed in the criteria matrix to insure a program's adjacency to windows. Priority Incorrect. The criteria matrix as a whole is meant to organize multiple priorities, so the program can be organized in a holistic manner.

The architect has chosen to utilize a criteria matrix in order to begin organizing the program. Which of the following spatial qualities would be listed in the criteria matrix?

Square footage Correct. Square footage of each program is typically listed in the criteria matrix to determine where it will fit. Public access Correct. Public access is typically listed to ensure both ease of access to the public programmatic requirements as well as privacy and security to other spaces. Adjacencies Correct. Adjacencies are typically listed so that similar and complimentary programs can be placed in proximity to each other. Daylight and view Correct. Both daylight and view are typically listed in the criteria matrix to insure a program's adjacency to windows. Priority Incorrect. The criteria matrix as a whole is meant to organize multiple priorities, so the program can be organized in a holistic manner

What does the Green Seal Standard GS-11 stands for?

Standard set of environmental, health, and performance requirements for sustainable paints and coating

A building is constructed with load-bearing concrete masonry units (CMU). The roof assembly is composed of open web steel joists at 4' on center with a concrete-filled steel deck, rigid insulation, and a membrane roof. Which part of the assembly is the diaphragm?

Steel deck Correct. The steel deck transfers the lateral forces to the walls and down to the foundation. The diaphragm is the horizontal structural element that transfers the lateral load to the vertical components of a structure. A continuos 3d member.

An architect is designing a shelter at a remote site in an arid climate where water is scarce. Clay found on the site will be used to create bricks for the primary construction material. The bricks will be bonded with a cement mortar. What type of admixture should the architect specify to reduce the amount of water required in the mortar mix?

Superplasticizers Correct. Superplasticizers improve the workability of a concrete or mortar mix at a given water-cement ratio. They can also be used to reduce the amount of mixing water needed for a desired workability. Accelerators Incorrect. Accelerators decrease the setting time of a mix, allowing concrete to be placed in winter with reduced risk of frost damage. However, while they speed up the cure time, accelerators do not protect the cured product over the long term against cracking induced by freeze-thaw cycles. Air-entraining agents Incorrect. Air-entraining agents disperse air in the form of small air bubbles throughout concrete, Portland cement paste, or mortar. This has a number of benefits to the cured product when used properly, including improved resistance to frost attack and deterioration. Surfactants Incorrect. Surfactants reduce the surface tension of the water in a concrete or mortar mix. This helps the cement particles and other additives in the mix to emulsify and disperse evenly.

An architect is working on the schematic design of a mixed-use, speculative office building for an urban infill site, where maximizing floor area for rental income is a primary project goal. When working to maximize the economically viable building size for the project, which of the following principles should the architect follow?

The architect should follow an iterative process to maximize the space for the rental area Correct. While working to maximize the economically viable building size, the architect would use an iterative process to maximize the space for the rental area. The architect should make assumptions regarding construction type and evaluate the relative cost-benefit analysis Correct. Before the owner can make a decision, the architect would make assumptions and evaluate the cost-benefit ratio for various construction types. The architect should figure out the maximum allowable height for the site, and then advise the client on which occupancy type would be appropriate Incorrect. The architect would determine the occupancy classification of the building first, and then determine the program area by either program needs or budget. A construction type will be determined after that, usually during schematic design.

An architect is evaluating the results of a feasibility study that their team just conducted for a proposed project. The total sources listed are $53,500,000, and the total uses listed are $56,750,000. Which of the following is true about the feasibility of the project based on this analysis?

The budget has a shortfall of $3.25 million Correct. The uses are $3.25 million more than the sources, indicating a shortfall, or deficit, of $3.25 million. The project has an excess of $3.25 million Incorrect. The scenario reflects a shortfall of $3.25 million because the uses (costs to complete the project) are $3.25 million more than the sources (the entity that will fund the project).

What is a bundled tube?

The bundled tube system involves, instead of one tube, several individual tubes interconnected to form a multi-cell tube. Together they work to resist the lateral loads and overturning moments. When the tubes fall within the building envelope, interior columns are positioned along their perimeters.

What shape offers both the best resistance to torsion and the least resistance to wind?

The circle offers the most resistance to torsion and the least resistance to wind.

An architect is designing a fitness center that will incorporate exercise rooms, changing rooms, and a lounge area. The owner would like to be efficient with space and have the largest exercise room be able to accommodate 45 people at a time. What is the minimum allowable area, in square feet, of an exercise room with an occupancy load of 45 people?

The correct answer is 2,250 square feet. According to IBC 2018 Table 1004.5, an exercise room must provide 50 square feet per occupant. If there are 45 occupants, then multiply 50 by 45 to find the required area of the room.

The architect is coordinating the sprinkler head layout with their plans in order to avoid all obstructions. What is the typical distribution pattern for a standard sprinkler head? Using the image below, drag the correct values onto the diagram.

The distribution pattern of a standard spray sprinkler is the cone of spray that each sprinkler head distributes when activated. The typical distribution pattern is a circle 1'6" below the head, with a radius of 4'-0" and a circle 4'-0" below the sprinkler head, with a radius of 8'-0"

The diagram below depicts a refrigeration cycle.

The expansion valve releases the pressure of the refrigerant, allowing it to expand. As the refrigerant travels through the evaporator coils, it cools the air. The compressor increases the pressure on the refrigerant, forcing it to heat. The heated refrigerant travels through the condensing coil and gives off hot air.

The floor area ratio of a building must be calculated in order to determine if it meets zoning regulations. What information should the architect gather?

The floor area of each story Correct. Knowing the floor area of each story is necessary to calculate the floor area ratio. The number of stories in the building Correct. Knowing the number of stories in the building is necessary to calculate the floor area ratio. The buildable land area of the site Correct. The goal of calculating the floor area ratio of a new building is to understand how big it can be on a given parcel of land, as defined by local zoning ordinances. The distance between exterior walls, including the width of the exterior walls Correct. Knowledge of the distance between exterior walls, including the width of the exterior walls, is necessary to calculate the floor area ratio.

What is the solar heat gain coefficient?

The fraction of solar radiation admitted through a window, door, or skylight -- either transmitted directly and/or absorbed, and subsequently released as heat inside a home. The lower the SHGC, the less solar heat it transmits and the greater its shading ability.

The architect has selected heavy timber framing for a new office building in a dense downtown area. In order to get the height of the building and of the spaces, the architect needs to preliminarily size the structural members. What should be the minimum dimension between the bottom of the beam and bottom of the girder?

The girder should be at least 2" deeper than the beam it supports. This allows for a char layer to occur on the girder while not compromising the integrity of its support of the beams.

volts x amps = watts

The metaphor is having a pipe with water and trying to fill a bucket. Volts are the pressure on which the water is being pushed out. Amps are the size of the pipe Watts is the power the thing you will work with

What iss waffle slab and maximum span?

The name waffle comes from the grid pattern created by the reinforcing ribs. Waffle slabs are preferred for spans greater than 40 feet (12 m), as they are much stronger than flat slabs, flat slabs with drop panels, two-way slabs, one-way slabs, and one-way joist slabs

What is one-way solid slab?

The one-way slab is a slab, which is supported by parallel walls or beams, and whose length to breadth ratio is equal to or greater than two and it bends in only one direction (spanning direction) while it is transferring the loads to the two supporting walls or beams, because of its geometry.

An architect is considering the benefits and limitations of entry systems for a new mixed-use building located in downtown Portland, OR. The street level of the building will contain various retail stores, two restaurants, and access to a business lobby which houses elevators for the offices on the upper floors of the building. The architect is reviewing the specifications for the installation of revolving doors at the building's main entrances. Which is true regarding revolving doors?

The practical capacity of each system is 25-35 people per minute Correct. Revolving doors can typically accommodate fewer people than sliding or swinging door systems. Depending on the size and speed of the revolving doors, approximately 25-35 people per minute can obtain entrance and/or egress. The doors can be accommodated on separate slab foundations Incorrect. Revolving doors must be mounted entirely on one slab foundation. If the doors span across multiple slabs and one slab is subject to settling, the doors may be off center and unable to revolve properly. Revolving doors contribute in full toward egress capacity Incorrect. Revolving doors are not considered to be egress options on life safety plans, and are therefore not able to contribute in full to egress capacity for a building. Some jurisdictions allow for revolving doors to count as 50% of legal exiting requirements, while others do not count them at all.

An architect is in the preliminary stage of design for a new high school building. The mechanical designer has suggested using a central HVAC all-water system. Which of the following is true regarding a central HVAC all-water system?

The system offers no humidification Correct. One major disadvantage to an all-water system is that it provides no humidification. The system offers no dehumidification Incorrect. This particuar HVAC system offers good dehumidification.

reverberation time

The time it takes for a sound produced in an enclosed space to decrease to 1/1,000th of its original pressure.

A design-build firm is researching Forest Stewardship Council (FSC) certification. Which of the following is accurate pertaiNing to wood certified by the FSC?

The wood was harvested without violating peoples rights The FSC is a global nonprofit dedicated to responsible forest management,. Indigenous Peoples Rights is Principles 3 of FSC certification. There is no requirement that a wood be locally sourced to be FSC-certified.

A design-build firm is researching Forest Stewardship Council (FSC) certification. Which of the following is accurate pertaining to wood certified by the FSC?

The wood was harvested without violating peoples' rights Correct. The FSC (Forest Stewardship Council) is a global nonprofit dedicated to responsible forest management. Indigenous Peoples' Rights is Principle 3 of FSC certification. The wood is locally sourced Incorrect. The FSC certifies that wood has come from responsibly managed forests. There is no requirement that a wood be locally sourced to be FSC-certified.

What is copper sulfate used for?

To help control algae growth in the water supply

Refer to the exhibit. An architect is designing a single-family house on a hill. The architect wants to minimize solar gain and maximize natural ventilation. Which one of the following locations for the house is most appropriate?

Top of a south-facing slope The table shows a hot and humid local climate. Therefore, the windy top is ideal for maximizing natural ventilation. Building on the south side of the hilltop also avoids the hot afternoon sun on the west side. Topic: Environmental Conditions & Context

A developer and their architect are planning to build a mixed-use building on a large suburban downtown site that is located two blocks from a commuter rail station. The building will contain high-density commercial and residential uses, with a pedestrian orientation and reduced parking requirements. Based on this information, what type of project is being proposed by the developer and the architect?

Transit-oriented development Correct. A transit-oriented development is a development typically located within a 10-minute walk from a transit station. To take advantage of the proximity to transit, the development is usually characterized by high density, mixed uses, pedestrian accessibility, and reduced parking requirements. Neighborhood center development Incorrect. Neighborhood centers provide more localized availability of goods and services for area residents within a neighborhood. They are developed as the focal point of the neighborhood and are usually located near the neighborhood's core. Besides residential and retail uses, a neighborhood center might contain professional offices, schools, restaurants, religious facilities, and public spaces.

What is a Vierendeel Truss?

Truss without triangles only right angles. Useful if you don't want angles truss components to interfere with windows, but for it to function as a truss, the connections at the top and bottom chords have to resist moment forces and are often beefy and expensive. Jenna building in New York achievers its cantilevers with two story concrete Vierendeel trusses. L

A new concrete parking structure with a capacity for 3,500 cars is being designed in the city center. It has been established that the structure will be built using rapid slip form construction. Which type of cement should the architect specify?

Type III Correct. Type III achieves strength values quicker than Type I, allowing the forms to be stripped sooner. Type I Incorrect. Type I is normal cement. This type is inappropriate for multi-level pours because of the time it takes to develop strength. Construction can proceed at a much faster pace for multi-level projects with other cement types. Type II Incorrect. Type II is moderate-sulfate-resistant cement, to be used when concrete is cast against soil that has moderate sulfate levels. Type IV Incorrect. Type IV is a low-heat-of-hydration cement, for use where the rate and amount of heat generated must be minimized. It develops strength at a slower rate than other types and is not suitable for the type of construction described in the question.

A decorative, non-load bearing masonry wall is being constructed in the lobby of an office building. This wall will be used as a thermal mass to passively heat the lobby.

Type O Is a low-to medium-strength mortar and can only be used for non load-bearing walls. It is the most cost-effective choice. Type N It is not as cost effective as type O Type M High-strength mortar used for severe weather exposures Type S medium to high strength mortar, and is commonly used in structural walls

An architect is working with their structural engineer on a mid-rise office building where they intend to use a braced framed system. In order to understand which members can be framed out of tensile materials, the architect will analyze the structure using only the lateral forces. Which of the following members will act in compression, tension, or neither? Using the image below, drag the correct designations onto the frame.

Under lateral loading conditions, this system will behave like a large truss to transmit the loads through a series of compression and tension axial members. The horizontal members, where the lateral loads are applied, will experience compression. The diagonal members that go down to the right, will also experience compression, since they are being pushed against. The diagonal members that go up to the right, will experience tension. The load is not directly applied to these members but the force pushing on the horizontal members that they are attached to causes pulling, or tension in these members. The members with no rotation do not experience truss action.

A family is consulting with an architect about designing a new custom residence. In addition to their current needs, they are also planning to have a cousin who uses a wheelchair come live with them sometime in the next five to ten years. The family needs the architect to design a home that will meet all of their needs now and in the future, without having to make adjustments when their cousin moves in. What principle of design should the architect follow for this project?

Universal Correct. Universal design is the creation of an environment that can be used to the greatest extent possible by all people regardless of their age, sex, size, or ability. It is a fundamental principle of good design that goes beyond just meeting the needs of a single individual. It makes accessibility an intrinsic part of the design of the home. Note: Adaptable Incorrect. Adaptable design looks at the ease of altering a room or space to meet the needs of individuals in the future. It does not address the needs of all of the users of the building like universal design does.

A client has purchased a plot of land and would like to build a prototype on it, requiring very little redesign. One major difference the mechanical team notices is that the original building is in a hot humid climate, and the new location is in a hot arid climate. This change will require an all new mechanical design. Which of the following cooling strategies are better suited for hot-arid climates than hot-humid climates?

Use night cooled mass Correct. Night cooled mass is more effective in hot-arid climates than in hot-humid climates due to larger daily temperature swings. Utilize evaporative cooling Correct. Evaporative cooling is used efficiently in hot-arid climates because of low humidity levels.

Which of the folllowing cooling strategies are better suited for hot-arid climates than hot-humit climates?

Use night-cooled mass Night-cooled mass is more effective in hot-arid climates due to larger daily temperature swings. Utilize evaporative cooling Evaporative cooling is used efficiently in hot-arid climates because of low humidity levels Use Cooling towers Cooling towers are a type of evaporative cooling and is a more efficient strategy for hot-arid climates

When do we prefer an eccentrically braced frame? (Steel)

Useful for earthquakes resists lateral seismic forces without buckling

What is a Swamp cooler?

Uses evaporation of water for cooling, with a fan, for very dry climates only.

A community board is reviewing the plans for the renovation of a multi-building public housing complex. The board wants to know which utilities/services are going to be affected by the renovation and how they will impact the design. Which utility and service pairs are correctly matched?

Utility: Electrical Correct. Electric service is a utility. Utilities can be public or private companies that provide water, sewer, electric, and gas. Utilities have infrastructure to build and maintain, and have rights to land use through easements. Service: Telecommunications Incorrect. Telecommunications is a utility. It is typically provided by private companies. Utility: Water Correct. Water service is a utility. Utilities can be public or private companies that provide water, sewer, electric, and gas. Utilities have infrastructure to build and maintain, and have rights to land use through easements. Utility: Sewer Correct. Sewer service is a utility. Utilities can be public or private companies that provide water, sewer, electric, and gas. Utilities have infrastructure to build and maintain, and have rights to land use through easements. Utility: Waste management Incorrect. Waste management is a service. Service: Gas Incorrect. Gas is a utility. Utilities can be public or private companies that provide water, sewer, electric, and gas. Utilities have infrastructure to build and maintain, and have rights to land use through easements.

A community board is reviewing the plans for the renovation of a multi-building public housing complex. The board wants to know which utilities/services are going to be affected by the renovation and how they will impact the design. Which utility and service pairs are correctly matched?

Utility: Electrical Correct. Electric service is a utility. Utilities can be public or private companies that provide water, sewer, electric, and gas. Utilities have infrastructure to build and maintain, and have rights to land use through easements. Utility: Sewer Correct. Sewer service is a utility. Utilities can be public or private companies that provide water, sewer, electric, and gas. Utilities have infrastructure to build and maintain, and have rights to land use through easements. Utility: Water Correct. Water service is a utility. Utilities can be public or private companies that provide water, sewer, electric, and gas. Utilities have infrastructure to build and maintain, and have rights to land use through easements. Utility: Waste management Incorrect. Waste management is a service.

A community board is reviewing the plans for the renovation of a multi-building public housing complex. The board wants to know which utilities/services are going to be affected by the renovation and how they will impact the design. Which utility and service pairs are correctly matched?

Utility: Sewer Correct. Sewer service is a utility. Utilities can be public or private companies that provide water, sewer, electric, and gas. Utilities have infrastructure to build and maintain, and have rights to land use through easements. Utility: Water Correct. Water service is a utility. Utilities can be public or private companies that provide water, sewer, electric, and gas. Utilities have infrastructure to build and maintain, and have rights to land use through easements. Utility: Electrical Correct. Electric service is a utility. Utilities can be public or private companies that provide water, sewer, electric, and gas. Utilities have infrastructure to build and maintain, and have rights to land use through easements Service: Telecommunications Incorrect. Telecommunications is a utility. It is typically provided by private companies.

What is a rotation fixed and translation free connection?

Utilize for seismic connection.

An architect is working on a new high-rise building and the client requests that sustainable materials and practices be used as much as possible. Which of the following are strategies the architect could employ to meet the client's goal? Check the four that apply.

Utilize prefabricated building elements Correct. By utilizing prefabricated building elements, there is less field-cutting of materials and job site waste can be decreased. Minimize field finishing of materials Correct. By finishing materials in the factory, the contaminants in the air on the job site can be reduced. These include volatile organic compounds (VOCs), dust from sanding, etc. Utilize smart technology Correct. Utilizing smart technology within the building is a great way to allow for the property to be more efficient in its operation. This technology includes occupancy sensors, smart thermostats, building automation systems, etc. Select products with high recycled content Correct. By selecting products with high recycled content, fewer raw materials need to be harvested. Specify high-VOC paint Incorrect. Specifying low-VOC (volatile organic compound) paint would help the client achieve their goal of a sustainable project.

An architect is designing a new nature center. A large, paved plaza in front of a multi-vehicle drop-off area is being planned at the main entrance of the project. What key considerations should the architect make when designing the plaza?

Utilizing a rigid paver system Correct. Rigid paver systems effectively shed water when installed correctly. Utilizing a rigid paving that collects water and then flows to a swale or stormwater wetland is a common solution to stormwater management. This system allows for a high level of control in terms of where water goes when it rains. Designing the plaza in a way that will allow runoff to be collected in a constructed stormwater wetland Correct. Constructed stormwater wetlands are designed to resemble natural environments and would be appropriate for the nature center. Utilizing a gravel for the plaza surface material Incorrect. Gravel is not a safe choice for finished plazas outside of large-scale attractions like nature centers, nor is it likely that the gravel will provide an accessible means of entry to the facility.

An architect is planning a new auto mechanic shop. The shop will have two repair bays, a storage area, and a small office. The building will be 1,000 square feet. The building is to be categorized as the least possible fire-resistive construction type. Which construction type is both appropriate and minimally fire-resistive?

V-B Correct. Type V-B construction is unprotected wood frame. This type has no fire rating requirement, making it the most combustible. It is appropriate for the auto shop because there are multiple occupancy types and the square footage is low enough to fit within the building area requirements of Type V-B construction. III-B Incorrect. Type III-B construction is unprotected combustible construction with brick or block walls. The exterior walls need to have a two-hour rating.

An architect is interested in incorporating the use of wind energy into an existing office building. During the site analysis, the architect studies the wind patterns to determine the prevailing winds. What should the architect use to review the prevailing wind directions?

Wind rose Correct. A wind rose depicts wind directions over a period of time and can be used to determine the prevailing wind directions in that location. Anemometer Incorrect. An anemometer is used to calculate wind speeds. This information would then be incorporated into a wind rose.

The owner of a proposed three-story office building would like their architect to design the building so that it will receive LEED Gold certification and meet their specific energy usage goals. While the architect cannot guarantee this, they explain to the owner that if the building is built in accordance with their drawings and specifications, it will be capable of obtaining the desired certification and meeting the energy usage goals. Which of the following factors that occur after design completion could affect the achievement of these sustainability goals? Check the four that apply.

Value engineering Correct. Value engineering by the owner or contractor to reduce costs during construction can have an impact on overall building and systems performance and certification. The architect should notify the owner and contractor of performance or certification issues that could be a result of proposed changes to the drawings and specifications. Building systems operation and maintenance Correct. When energy usage does not meet specific goals, it is often caused by the owner not operating the building systems as they were designed to be run. Contractor documentation Correct. The contractor must accurately compile the required documentation during construction that will be submitted for LEED certification. If the contractor fails to provide this information, the building may not be certified. Certification approval Correct. LEED certification is not guaranteed. Various elements in the final construction and submitted documentation could affect whether certification will be awarded, or what the level of certification will be. Shop drawing approvals Incorrect. Shop drawing approvals by the architect should not affect the ability to meet sustainability goals, if the submittals meet the project specifications. If a submittal is made that does not meet specification or performance criteria, the architect should make note of the fact and return the shop drawing unapproved.

A developer has hired a team of designers to lay out a residential development next to a freeway. The designers are exploring solutions to reduce the presence of the freeway for the occupants of the development. What should the architect incorporate into the space between the development and the freeway?

Vegetated berm Correct. A planted berm can separate incompatible uses, provide a visual distraction, and create a feeling of expansiveness. A berm can help block sound as well. Masonry barrier wall Incorrect. While the wall might provide some visual and acoustic screening, the presence of the freeway would still be felt. Sounds would still reflect off the wall, and the wall might create pockets of low air quality next to the freeway.

Examples of areas to utilize a dry pipe system?

Ventilated attic spaces Freezer Storage Rooms A dry pipe system is needed to prevent the pipes from freezing

An architect is selecting a sprinkler system for a building in a cold climate. For which of the following should the sprinkler system be used?

Ventilated attic spaces Freezer storage room The illustration shows a dry pipe system. In both spaces, a dry pipe system is needed to prevent the pipes from freezing.

An architect is specifying a geo-exchange heat pump system for an office building. The building is on a site that has a significant concentration of groundwater and is conducive to well drilling. The owner wants the heat pump system that is the most cost-effective and energy-efficient. Which heat pump system should the architect specify?

Vertical closed loop For ground that is conducive to well drilling, the deep ground will be warmer in the winter and cooler in the summer than ground closer to the surface. Vertical systems are most efficient in ground that is conducive to well drilling. Closed loop systems use much less pumping energy than open loop systems.

An architect is interested in incorporating the use of wind energy into an existing office building. During the site analysis, the architect studies the wind patterns to determine the prevailing winds. What should the architect use to review the prevailing wind directions?

Wind rose Correct. A wind rose depicts wind directions over a period of time and can be used to determine the prevailing wind directions in that location. Wind sock Incorrect. A wind sock is used to measure wind speed and direction at a given point in time. A wind rose would be more appropriate because it depicts wind speeds and directions over an extended period of time.

An architect is performing a code review for a new school building. During the review, the architect reviews the International Energy Conservation Code in addition to the International Building Code. Which of the following items are controlled by the International Energy Conservation Code (IECC)?

Wall insulation requirements Correct. The IECC specifies the wall insulation requirements. Lighting controls Correct. The IECC specifies lighting control requirements to maximize energy efficiency. Electrical power consumption Correct. The IECC specifies the maximum electrical power consumption for different features such as electric water heaters and lighting. Total building performance requirements Correct. The IECC denotes the requirements for total building performance. Glazing thickness Incorrect. While the IECC specifies thermal performance criteria for glazing, the overall construction of the glazing assembly is controlled by the IBC. Ventilation requirements Incorrect. Ventilation requirements are defined in the International Mechanical Code. However, the IECC does specify the efficiency requirements for various pieces of mechanical equipment.

An architect is evaluating materials for the structural system of a new medical office building. Since the project is only going to be one story, the architect is considering both light gauge metal framing and wood framing for the project. What are some of the benefits to using light gauge metal framing instead of wood framing? Check the four that apply.

Water resistance Correct. Metal studs typically have a galvanized coating applied to them in the factory, giving the studs a greater water resistance than non-treated wood studs. Larger load capacity Correct. Steel studs have a higher load capacity compared to traditional wood studs. In addition, studs of the same depth are available in a range of steel thicknesses, which allows for heavier loads within a wall to be carried while still maintaining a consistent depth. Noncombustible Correct. While wood can be treated with a fire-retardant coating, metal studs are inherently noncombustible without additional treatments. Increased construction efficiency Correct. Metal studs can be delivered with pre-punched web holes, allowing for faster completion of utility rough-ins.

A structural engineer has told an architect that the total pressure on a retaining wall cannot exceed 1500 plf. The concrete wall has a total length of 40 ft and is adjacent to a parking lot. The pressure on the wall has been calculated to be 30 lbs per cubic foot. Use the formula P = 0.5 (p) h^2. How high can the architect make the wall?

We will use the formula P = 0.5 (p) h^2, where the variables are defined as follows: P = total pressure (1500 plf) p = media pressure (30 lbs/ft^3) h = height Plugging in the numbers that are known, we get: 1500 plf = 0.5 (30 lbs/ft^3) (h^2) Using algebra to solve for h, we find the answer:h = 10 ft

An architect is designing a new boat house by a lake. The site slopes down from the boat house towards the lake with a significant 2' drop in one location that is being shored through the use of a 2' high brick retaining wall. Which of the following components may be omitted from the design of the retaining wall?

Weep holes and drainage pipe Correct. For retaining walls 2'-0" tall or less, seep holes and drainage pipes are not required. Concrete footing located below the frost line Incorrect. Despite the wall being diminutive, a concrete footing would still be required to resist the overturning force of the soils above.

An architect is calculating the occupancy load for a building. Which of the following requirements are dependent on the occupancy load?

Width of egress stairs Correct. Occupancy load is used to determine the minimum required stair width. The direction of door swings Correct. Occupancy load is used to determine door swings. Number of exits Correct. Occupancy load is used to determine number of required exits. Maximum building height and area Incorrect. Zoning regulations and type of construction, in conjunction with occupancy use group, determines building height and area limitations.

What should the architect use to review the prevailing wind directions?

Wind Rose Note that an ANEMOMETER is used to calculate wind speeds. This information would then be incorporated into ta wind rose

The owner has expressed interest in creating artist studios as allowed by zoning. There is concern regarding the adequacy of the existing structural system and its ability to carry loads from heavy equipment. The dead load is assumed to be 50 psf at interior floors, and dead and live loads on the roof are assumed to be 30 psf each. What is the total load that must be carried by a freestanding ground-floor column in the east building?

the answer is 131040 To determine the total load on a ground-floor column, first calculate the tributary area. The tributary area of the freestanding column is found by measuring the distance to each of the nearest columns, and dividing that distance in half. The resultant rectangle (in this example, 26' x 18') is the tributary area of the column. Once the tributary area is determined, the dead and live loads at each floor are multiplied by this value. Add the total calculated at each floor which sits above the column to determine the total load. Tributary Area = 468 sf (26' x 18')Roof load = 60 psf (30psf dead load + 30psf live load - refer to question) 3rd floor load = 110 psf (50psf dead load + 60psf live load - refer to code excerpt) 2nd floor load = 110 psf (50psf dead load + 60psf live load - refer to code excerpt) 468 sf x (60 lbs + 110 lbs + 110 lbs) = 131,040 lbs

An architect is calculating the average expected indoor illumination level in a room from daylighting with an overcast winter sky.The following information is known: The daylight factor in the room is 2% The lowest average outdoor illumination level from an overcast winter sky is 1000 footcandles The size of the room is 16 feet by 18 feet Based on these conditions, what is the average expected indoor illumination level? Reference formula:daylight factor = interior illuminance / exterior illuminance to calculate.

the answer is 20 Daylight factor is defined as the ratio of exterior illuminance to interior illuminance. Use the equation: daylight factor = interior illuminance/exterior illuminance. Rearrange the equation to solve for interior illuminance: interior illuminance = daylight factor x exterior illumination. Solve for interior illuminance.0.02 x 1000 = 20 footcandles

The fourth floor of a proposed apartment building has an occupant load of 383. Four means of egress doors will be provided. Calculate the required minimum width for each means of egress door opening. Use 0.2" of egress width per occupant and round up to the nearest inch

the answer is 32 Take the occupant load and multiply it by the egress width per occupant:383 x 0.2" = 76.6"Now divide among four egress doors:76.6"/4 = 19.15" Lastly, round up to 20" However, International Building Code (IBC) 1010.1.1 states, "The required capacity of each door opening shall be sufficient for the occupant load thereof and shall provide a minimum clear opening width of 32".


Related study sets

Percy Jackson The Lightning Thief End of Novel Test

View Set

Psych 1.1.1: What is Psychology?

View Set

Chapter 10 Terms Social Psychology and Human Nature 3e

View Set

Chapter 20: Genomics 1: Analysis of DNA

View Set

7th Grade SS Chapter 5 The Decline of Feudalism

View Set

Praxis 2: content knowledge (5018)

View Set

electrostatics (multiple choice from the packet)

View Set

Quiz 8: Ch 23-The Digestive System

View Set

Segment 5 - Federal Laws Governing Mortgage Practice

View Set

Fr1 2.1 #2 Which subject pronoun replaces...?

View Set

ECON 320 Chapter 6 Connect Questions Spring 2022

View Set